102
Challenge

Head and Neck Imaging - Case Review Series - Challenge

Embed Size (px)

Citation preview

Page 1: Head and Neck Imaging - Case Review Series - Challenge

Challenge

Page 2: Head and Neck Imaging - Case Review Series - Challenge
Page 3: Head and Neck Imaging - Case Review Series - Challenge

307

C A S E 1 5 1

History: An 8-month-old with bilateral hearing loss being evaluated for cochlear implant (CI).

1. Which ear is a better candidate for a CI? A. Right B. Left

2. Which of the following CT findings best predicts the absence of the cochlear nerve (cochlear nerve deficiency)? A. Anomalous cochlea B. Anomalous vestibule C. Narrow IAC D. Narrow cochlear nerve aperture

3. Which of the following constitutes a contraindication for CI? A. Michel’s anomaly B. Mondini malformation C. Incomplete partition type I D. Common cavity E. Vestibular dysplasia

4. All of the following statements regarding cochlear implants are true EXCEPT: A. Hearing and speech outcome with bilateral CIs is

better than with a unilateral implant in the con-genitally deaf.

B. In the congenitally deaf, CI is ideally performed at age 4 after cochlear development is complete.

C. With special precautions, patients with CIs can have MRI scans.

D. NF2 patients with bilateral schwannomas can benefit from CI.

E. When cochlear nerve deficiency is present, audi-tory brain stem implantation may be an option.

Page 4: Head and Neck Imaging - Case Review Series - Challenge

308

A N S W E R S

C A S E 1 5 1

Cochlear Nerve Deficiency1. B

2. D

3. A

4. B

ReferenceAdunka OF, Jewells V, Buchman CA. Value of computed

tomography in the evaluation of children with cochlear nerve deficiency. Otol Neurotol. 2007 Aug;28(5):597–604.

Cross-ReferenceNeuroradiology: THE REQUISITES, 3rd ed, p 404.

CommentAbsence of the cochlear nerve, or cochlear nerve defi-ciency (CND), is a critical finding for patients with hearing loss being evaluated for CI placement. High-resolution T2W MRI allows direct identification of the cochlear, vestibular, and facial nerves in the IAC (see the figure). Many believe that an absent cochlear nerve per MRI should exclude that ear from cochlear implantation, although according to others some small nerve fibers below the resolution of MRI may be present. In the past, it was believed that if the cochlea itself is morphologi-cally intact there must be a cochlear nerve; today, we not infrequently see an absent nerve with normal-appearing cochlea, which makes MRI a much better test than CT for the initial evaluation of these children. A narrow IAC (<2.5 mm) identified on CT is associated with CND, but CND does occur in patients with a normal-size IAC and a normal cochlear nerve may be present in a narrow IAC. When there is narrowing of the IAC, MRI becomes difficult to interpret because of a paucity or lack of CSF around the nerves. Stenosis or closure of the cochlear nerve aperture at the base of the cochlea is a much better predictor of CND than narrow IAC; if the cochlear aperture is closed, there is no cochlear nerve. A stenotic cochlear nerve aperture also strongly corre-lates with CND. CT is the current method of choice for the evaluation of cochlear aperture.

Notes

Page 5: Head and Neck Imaging - Case Review Series - Challenge

309

C A S E 1 5 2

History: Visual loss

1. Which of the following should be included in the dif-ferential diagnosis for this lesion of the globe? (Choose all that apply.) A. Retinal metastasis B. Retinal hemangioblastoma C. Choroidal angioma D. Melanoma

2. Which of the following orbital anomalies is most closely associated with neurofibromatosis type 1 (NF1)? A. Lisch nodule B. Hemangioblastoma C. Choroidal angioma D. Retinal astrocytoma E. Arteriovenous malformation (AVM)

3. Which of the following orbital anomalies is most closely associated with tuberous sclerosis? A. Lisch nodule B. Hemangioblastoma C. Choroidal angioma D. Retinal astrocytoma E. AVM

4. Which of the following orbital anomalies is most closely associated with von Hippel Lindau disease (VHL)? A. Lisch nodule B. Hemangioblastoma C. Choroidal angioma D. Retinal astrocytoma E. AVM

Page 6: Head and Neck Imaging - Case Review Series - Challenge

310

A N S W E R S

C A S E 1 5 2

Retinal Hemangioblastoma1. A, B, C, and D

2. A

3. D

4. B

ReferenceChew EY. Ocular manifestations of von Hippel-Lindau

disease: clinical and genetic investigations. Trans Am Ophthalmol Soc. 2005;103:495–511.

Cross-ReferenceNeuroradiology: THE REQUISITES, 3rd ed, p 311.

CommentThe phakomatoses have common elements of skin, eye, and CNS disorders. The orbital manifestations of NF1 are manifold but include Lisch nodules of the iris, sphenoid wing dysplasia, and optic pathway/nerve gliomas. Plexi-form neurofibromas may also affect the trigeminal nerve and have orbital manifestations, and malignant transfor-mation is always a danger. Neurofibromas may affect all parts of the orbit.

NF2 can have meningiomas and schwannomas that affect the orbit.

VHL classically shows retinal hemangioblastomas of the globe, as in this case (see the figures). Retinal capil-lary hemangiomas are also prevalent in this disorder. Metastases from renal cell carcinoma to the orbit may also occur. Between 50% and 65% of patients with VHL have ocular involvement, but visual acuity is usually not affected. Partial deletion, missense, and nonsense muta-tions of VHL-related genes on chromosome 3 predispose to retinal hemangioblastomas.

Sturge-Weber syndrome is associated with choroidal hemangiomas in 40% of cases. This is not the choroid plexus but the ocular membrane tumor. They obvi-ously also will have the port-wine nevus flammeus, which may affect the skin of the eye. Glaucoma is another known association.

Tuberous sclerosis most commonly shows retinal astrocytomas. Fifty percent of patients with tuberous sclerosis develop at least one retinal astrocytoma in one eye at some point in the course of their disease.

Wyburn-Mason syndrome may have AVMs that can affect all parts of the orbit, including the globe, soft tissue, and bone. Retinal AVMs may develop with increas-ing age.

Notes

Page 7: Head and Neck Imaging - Case Review Series - Challenge

311

C A S E 1 5 3

History: A 10-month-old boy presents with a neck mass that appeared and grew in 2 weeks.

1. Based on the imaging features and clinical presenta-tion, is this mass more likely to be benign or malignant? A. Benign B. Malignant

2. Biopsy revealed a benign mass. Which of the follow-ing benign entities is more likely? A. Myxoma B. Nodular fasciitis C. Dermoid D. Schwannoma E. Fibroma

3. Which of the following represents the most common location of nodular fasciitis? A. Head and neck B. Upper extremities C. Lower extremities D. Retroperitoneum E. Trunk

4. Nodular fasciitis has a tendency to involve the deep neck spaces. True or false?

Page 8: Head and Neck Imaging - Case Review Series - Challenge

312

A N S W E R S

C A S E 1 5 3

Nodular Fasciitis1. B

2. B

3. B

4. False

ReferenceKim ST, Kim HJ, Park SW, et al. Nodular fasciitis in the

head and neck: CT and MR imaging findings. AJNR Am J Neuroradiol. 2005 Nov–Dec;26(10):2617–2623.

Cross-ReferenceNeuroradiology: THE REQUISITES, 3rd ed, p 503.

CommentNodular fasciitis is a benign, idiopathic, reactive prolif-eration of myofibroblasts found in the subcutaneous fascia and presents as a rapidly growing mass, leading to it frequently being mistaken for a sarcoma. It occurs mostly in the subcutaneous fat and fascia of the upper extremities in adults. Fifteen percent of cases occur in the subcutaneous tissues of the head and neck, half of which are found in children, showing a predilection for this anatomic site in the younger population. When seen in children, it may be confused with other more common soft-tissue tumors, such as hemangioma, venous malfor-mation, fibroma, myxoma, desmoid tumor (fibromato-sis), sarcoma, lymphoma, and parotid tumor. The etiology is unclear; many believe that it is a non-neoplastic response to injury because a history of trauma can be elicited from some patients. The subcutaneous location of the tumor is the most important clue to the diagnosis; when they are large, they infiltrate the adjacent neck muscles, as seen in this case, mimicking a malignant tumor (see the figures). The histologic features of nodular fasciitis are variable; some tumors show marked cellular-ity and increased T2W signal and some show marked fibrous component, which presents as a T2 hypointense mass. Enhancement with contrast is usually strong. Cystic areas may be seen in some cases of nodular fasci-itis. Once the benign diagnosis is confirmed with biopsy, the treatment consists of simple resection. Recurrence is rare.

Notes

Page 9: Head and Neck Imaging - Case Review Series - Challenge

313

C A S E 1 5 4

History: Bilateral puffiness in the neck

1. Which of the following disorders is included in the differential diagnosis? (Choose all that apply.) A. Lipoma B. Metastasis C. Cushing’s disease D. Madelung’s disease E. Liposarcoma

2. Which of the following represent the characteristic features of Madelung’s disease? A. Symmetry B. Orbit location favored C. Encapsulated fat D. Increased risk of liposarcoma E. Favors females over males

3. Which of the following are NOT features of Made-lung’s disease? (Choose all that apply.) A. More common in drinkers B. Painful C. More common in Europeans than Americans D. Usually vascular

4. Peripheral neuropathies are a feature of Madelung’s disease. True or false?

Page 10: Head and Neck Imaging - Case Review Series - Challenge

314

A N S W E R S

C A S E 1 5 4

Madelung’s Disease1. A, C, D, and E

2. A

3. B

4. True

ReferenceRamos S, Pinheiro S, Diogo C, et al. Madelung disease: a

not-so-rare disorder. Ann Plast Surg. 2010 Jan;64(1):122–124.

Cross-ReferenceNeuroradiology: THE REQUISITES, 3rd ed, p 494.

CommentMadelung’s disease is an idiopathic symmetric lipomato-sis of the neck, also known as Launois-Bensaude adeno-lipomatosis, first described by Otto Madelung. The fatty infiltration is usually distributed in the neck and across the shoulders (horse collar) and hips (see the figure). It is found most commonly in countries bordering the Mediterranean Sea. There is an association with alcohol-ism, and the disease occurs more commonly in men than women, in a greater than 8:1 ratio. Symptoms may be due to peripheral neuropathies, but the entity is usually painless. Rapid growth may occur over weeks to months followed by stabilization of the deformity.

HIV-positive patients taking protease inhibitors may develop a Madelung-like condition with diffuse prolifera-tion of the subcutaneous fat.

Treatment with liposuction is preferred over dermoli-pectomy. This is usually carried out for the cosmetic deformity rather than neurologic complaints.

Notes

Page 11: Head and Neck Imaging - Case Review Series - Challenge

315

C A S E 1 5 5

History: A 71-year-old with progressively worsening hoarseness and breathing difficulty

1. Which of the following disorders should be included in the differential diagnosis? (Choose all that apply.) A. Wegener’s granulomatosis B. Lymphoma C. Rhinoscleroma D. Amyloidosis E. Mucopolysaccharidosis

2. What is the upper limit of the thickness of mucosa/submucosa in the subglottis/trachea on imaging studies? A. 0.5 mm B. 1 mm C. 1.5 mm D. 3 mm

3. Which of the following represents the most impor-tant element of the patient’s history in the differential diagnosis of this lesion? A. Presence of a systemic disease B. Tracheal intubation C. Ethnicity, travel history D. Smoking

4. All of the following statements regarding aerodiges-tive system amyloidosis are true EXCEPT: A. It is associated with systemic amyloidosis. B. The larynx is the most common site. C. The subglottis is the most common subsite of the

larynx to be involved with amyloidosis. D. Laryngeal amyloid deposits frequently show

decreased T2 signal on MRI.

Page 12: Head and Neck Imaging - Case Review Series - Challenge

316

A N S W E R S

C A S E 1 5 5

Amyloidosis1. A, B, C, and D

2. A

3. B

4. A

ReferencePenner CR, Muller S. Head and neck amyloidosis:

a clinicopathologic study of 15 cases. Oral Oncol. 2006 Apr;42(4):421–429. Epub 20 Feb 2006.

Cross-ReferenceNeuroradiology: THE REQUISITES, 3rd ed, p 455.

CommentWithin the cricoid and tracheal cartilage ring, airway mucosa directly abuts the cartilage and there is no visible soft tissue between the cartilage and air column. Any symmetric or asymmetric soft-tissue thickening within the subglottic airway should be regarded as abnormal (see the figure). By far the most common cause of sub-glottic stenosis is previous tracheal intubation. This usually presents on imaging as circumferential narrow-ing of the subglottic airway. Although laryngeal and hypopharyngeal cancer can invade the subglottic airway, these usually appear as eccentric mass lesions. Heman-gioma of the subglottic region, a relatively common cause of laryngeal stenosis in children, also usually pres-ents as an eccentric submucosal mass. In addition to prior intubation, possible causes of circumferential subglottic stenosis include Wegener’s granulomatosis, rhinoscleroma, amyloidosis, and lymphoma. Subglottic stenosis is found in 10% to 20% of patients with Wegener’s granulomatosis, and at times, it may be the only indication of disease. Rhinoscleroma is a rare chronic progressive granulomatous disease of the respiratory tract that most frequently affects the nose, paranasal sinuses, and larynx. The causal organism is Klebsiella rhinoscleromatis. Rhinoscleroma is seen more frequently in Africa and the Middle East. Amyloid deposition in the upper aerodigestive tract is usually an isolated phenomenon (as in this case) rather than a mani-festation of systemic amyloidosis, which occurs only in a minority of these patients. Some may develop systemic amyloidosis in the following years after the upper aerodi-gestive tract amyloidosis is diagnosed, however. Although isolated amyloid deposition can involve all anatomic regions in the upper aerodigestive tract, the larynx is the most common site, and the subglottis is the most common subsite in the larynx. On imaging, amyloid deposition can show diffuse circumferential involve-ment of the airway, but it can also attain focal nodular

appearance, mimicking other mass lesions. Amyloidosis is said to be associated with decreased T2 signal, although this may also be the case with lymphoma and granulo-matous lesions, and differentiation of these entities with imaging is not possible.

Differential Diagnosis of Circumferential Subglot-tic StenosisPrevious tracheal intubationRhinoscleromaAmyloidosisWegener’s granulomatosisLymphoma

Notes

Page 13: Head and Neck Imaging - Case Review Series - Challenge

317

C A S E 1 5 6

History: Hearing loss in an infant

1. Of the following differential diagnoses, which best describes this patient’s disorder? A. Cock’s deformity B. Mondini malformation C. Michel’s dysplasia D. Pendred syndrome E. None of the above

2. Which of the following cochleovestibular dysplasias is the last to occur embryologically? A. Common cavity B. Michel’s dysplasia C. Mondini malformation D. Hypoplasia of the lateral semicircular canal

3. Which of the following percentages represents the incidence of middle ear deformity in a patient with a Mondini malformation? A. <25% B. 26–50% C. 51–75% D. >76%

4. Which of the following is the most common inner ear congenital disorder? A. Mondini malformation B. External canal atresia C. Superior semicircular canal dehiscence

syndrome D. Enlarged vestibular aqueduct syndrome

Page 14: Head and Neck Imaging - Case Review Series - Challenge

318

A N S W E R S

C A S E 1 5 6

Otic Dysplasia/Incomplete Partition Type III1. E

2. D

3. A

4. D

ReferenceGoldfeld M, Glaser B, Nassir E, et al. CT of the ear

in Pendred syndrome. Radiology. 2005 May;235:537–540.

Cross-ReferenceNeuroradiology: THE REQUISITES, 3rd ed, p 405.

CommentThis case is a rare example of cochlear maldevelopment that presents with absence of the modiolus and bulbous dilatation of the cochlea, which can be best categorized as incomplete partition type III (IP-III), a recent addition to the cochleovestibular dysplasia classifications. The vestibule appears to be separate from the deformed cochlea. Therefore, the case is not one of a common cavity. The turns of the cochlea can be appreciated, but they are enlarged and dysmorphic. The lateral aspect of the IAC is markedly dilated (see the figures). On the right side one can just make out the endolymphatic sac/vestibular aqueduct, which is not enlarged.

Two different types of incomplete partition have clas-sically been described: IP-I (unpartitioned cochlea, cystic cochleovestibular malformation) and IP-II (incompletely partitioned cochlea, the classic Mondini malformation). In IP-I, the cochlea is “empty” and lacks the entire modi-olus and interscalar septa, resulting in a cystic appear-ance. The vestibule is grossly dilated without a vestibular aqueduct. In IP-II, the cochlear deformity has three com-ponents: (1) a cystic apex with a normal basal turn of the cochlea, (2) a dilated vestibule, and (3) a large ves-tibular aqueduct. The degree of dysplasia is milder in IP-II than in IP-I.

From very early in embryology, the fetus may have arrested development of the otic structures. There may be (1) complete absence of the cochlea and vestibule, as in Michel’s labyrinthine aplasia (3rd week of gesta-tion), (2) a common cavity where cochlea and vestibule fuse to one (4th week of gestation), (3) isolated cochlear aplasia (5th week of gestation), (4) cochlear hypoplasia (6th week of gestation), (5) incomplete partition defor-mity with abnormal scala/modiolus (6th week of gesta-tion), (6) abnormal spiralization of the cochlea in the Mondini malformation (7th week of gestation). In each of these cases the infant will have hearing loss; it is criti-cal to ensure that there is a nerve (usually spotted on

high-resolution CISS images through the IAC) present that would enable hearing if a corrective procedure or implant is contemplated. The location of the facial nerve also is important to know at all times.

Enlargement of the endolymphatic sac in association with a large vestibular aqueduct is present in about 20% of Mondini malformations.

Pendred syndrome combines congenital hypothyroid-ism and goiter with congenital hearing loss due to inner ear malformation on both sides, including absence of the modiolus (100%), enlarged vestibule (100%), absent interscalar septum (75%), and enlarged vestibular aque-ducts (80%). The abnormal gene, SLC26A4/PDS, found on chromosome 7, leads to abnormal synthesis of the protein pendrin, a sulfate transporter.

It must be remembered that a large number of con-genital conditions that go with sensorineural hearing loss have no radiologic manifestation but distinct clinical or genetic features. With improving ability to image the fine structures of the labyrinth, some of these will come to the focus of radiologists with perhaps newer classifica-tion schemes emerging.

Notes

Page 15: Head and Neck Imaging - Case Review Series - Challenge

319

C A S E 1 5 7

History: A 52-year-old woman with hoarseness.

1. Is this tumor in the post-styloid or prestyloid para-pharyngeal space? A. Post-styloid B. Prestyloid

2. Which of the following illustrates the order of fre-quency of jugular canal tumors from most common to least common? A. Schwannoma–glomus tumor–meningioma–

metastasis B. Glomus tumor–schwannoma–meningioma–

metastasis C. Glomus tumor–meningioma–schwannoma–

metastasis D. Schwannoma–meningioma–glomus tumor–

metastasis E. Schwannoma–glomus tumor–metastasis–

meningioma

3. All of the following pairs of imaging findings and tumors match EXCEPT: A. Intratumoral flow voids; glomus tumor B. Intratumoral cysts; schwannoma C. Lytic destruction of the skull base; glomus tumor D. Smooth expansion of the jugular canal;

schwannoma E. Sclerosis of the skull base; meningioma F. Calcification; schwannoma

4. The internal carotid artery is considered to be infil-trated when the tumor surrounds it more than: A. 90 degrees B. 180 degrees C. 270 degrees D. 359 degrees

Page 16: Head and Neck Imaging - Case Review Series - Challenge

320

A N S W E R S

C A S E 1 5 7

Meningioma of the Post-styloid Space1. A

2. B

3. F

4. C

ReferenceRamina R, Maniglia JJ, Fernandes YB, et al. Jugular

foramen tumors: diagnosis and treatment. Neurosurg Focus. 2004 Aug 15;17(2):E5.

Cross-ReferenceNeuroradiology: THE REQUISITES, 3rd ed, p 384.

CommentMeningiomas of the jugular canal presenting as a neck mass are exceedingly rare and mimic glomus tumors because of their vascular nature. Glomus tumors make up about 80% of the mass lesions that involve the jugular foramen. Schwannomas constitute the majority of the rest. Meningioma, chordoma, chondrosarcoma, and metastasis occurring in the jugular canal are rare. Most glomus tumors of the jugular canal extend both intracranially and extracranially, whereas most menin-giomas are intracranial with secondary extension into the jugular foramen. Schwannomas can be primarily extracranial, intracranial, or transcranial. The pattern of bone involvement is helpful in differentiating glomus tumors from others; sclerosis of the occipital bone is indicative of meningioma, as demonstrated in this case (see the figures). Glomus tumors usually cause a lytic destruction of the bone, whereas schwannomas are asso-ciated with smooth remodeling of the canal walls.

Notes

Page 17: Head and Neck Imaging - Case Review Series - Challenge

321

C A S E 1 5 8

History: Jaw pain in a woman being treated for meta-static breast cancer

1. Which of the following are the possible causes of the bone lesion in this patient given the history provided? (Choose all that apply.) A. Pathologic fracture due to steroid administration B. Metastasis C. Osteonecrosis from bisphosphonate

administration D. Osteomyelitis due to immunocompromised state E. Torus mandibuli

2. For which of the following conditions are bisphos-phonates administered? A. Breast cancer bone metastases B. Paget’s disease C. Osteoporosis D. Hypercalcemia E. All of the above

3. Which of the following represents the incidence of jaw necrosis in a patient with bone metastases being treated with bisphosphonates? A. <25% B. 26–50% C. 51–75% D. >76% E. All of the above

4. Which of the following represents the approximate ratio of involvement of the mandible versus the maxilla in bisphosphonate-induced osteonecrosis? A. 10:1 in favor of the mandible B. 10:1 in favor of the maxilla C. 3:1 in favor of the maxilla D. 3:1 in favor of the mandible

Page 18: Head and Neck Imaging - Case Review Series - Challenge

322

A N S W E R S

C A S E 1 5 8

Bisphosphonate-related Osteonecrosis1. A, B, C, and D

2. E

3. A

4. D

ReferenceThumbigere-Math V, Sabino MC, Gopalakrishnan R, et al.

Bisphosphonate-related osteonecrosis of the jaw: clini-cal features, risk factors, management, and treatment outcomes of 26 patients. J Oral Maxillofac Surg. 2009 Sep;67(9):1904–1913.

Cross-ReferenceNeuroradiology: THE REQUISITES, 3rd ed, pp 450–

451.

CommentOsteonecrosis of the maxilla is one potential complica-tion of the use of bisphosphonates to treat the bone metastases of breast/lung/prostate cancer and myeloma. These agents are also used for hypercalcemia, Paget’s disease, and generalized osteoporosis. The onset of osteonecrosis is shorter (2–3 years) after use for cancer treatment than for osteoporosis (6 years). The risk factors for development of osteonecrosis include female sex, recent dental extractions, IV administration, high dose, and age older than 60 years.

Imaging features reveal a mixed pattern of both osteo-lytic lesions and sclerotic regions in the mandible or maxilla, but lucent lesions predominate on CT (see the figures). Periosteal reaction is variable but more com-monly seen than not. On T1W MRI the normal marrow signal is replaced with hypointense signal. Enhancement in the cortex and subcortical bone is present, and the enhancement may involve the adjacent muscles of mastication.

This complication of therapy is very difficult to treat and withdrawal of the agent does not necessarily lead to reversal of the process. Recommendations from the Canadian Association of Oral and Maxillofacial Surgeons include the following:

1. Dental examination and radiographs before the initia-tion of IV bisphosphonate therapy

2. Completion of urgent surgical dental procedures before the initiation of high-dose bisphosphonate therapy and nonurgent procedures 3 to 6 months after cessation of bisphosphonate therapy

3. Cessation of smoking, limiting alcohol intake, and maintaining good oral hygiene

4. If osteonecrosis occurs, initiate supportive care, manage pain, treat secondary infection, remove necrotic debris and sequestrum

Notes

Page 19: Head and Neck Imaging - Case Review Series - Challenge

323

C A S E 1 5 9

History: A 4-month-old boy presents with a neck mass identified during a workup for torticollis.

1. Which of the following conditions should be included in the differential diagnosis? (Choose all that apply.) A. Neuroblastoma B. Rhabdomyosarcoma C. Ectopic thymus D. Dermoid/epidermoid E. Thyroglossal duct cyst

2. Which of the following imaging features suggests a diagnosis other than torticollis? A. Mass next to the sternocleidomastoid muscle

(SCM) B. Heterogeneous SCM C. Focal enlargement of SCM D. Diffuse enlargement of SCM E. Lack of adenopathy

3. Which of the following locations is the most common site of ectopic thymic tissue in the neck? A. Paraparyngeal space B. Masticator space C. Visceral space D. Parotid space E. Floor of the mouth

4. Ninety percent of ectopic thymic lesions are cystic. True or false?

Page 20: Head and Neck Imaging - Case Review Series - Challenge

324

A N S W E R S

C A S E 1 5 9

Ectopic Thymus1. A, B, and C

2. A

3. C

4. True

ReferencePark JJ, Kim JW, Kim JP, et al. Two cases of ectopic

cervical thymus: case reports and a review of the lit-erature. Auris Nasus Larynx. 2006 Mar;33(1):101–105. Epub 21 Feb 2006.

Cross-ReferenceNeuroradiology: THE REQUISITES, 3rd ed, p 495.

CommentEctopic thymus tissue in the neck is a rare cause of neck mass in children. A thymic remnant can present as a thymic cyst or less frequently as solid thymic tissue. Thymic remnants in the neck have been reported in 9% of autopsies. The most common location is about the thyroid gland, but it can occur along the path of descent of the primordial thymus from the angle of the mandible to the mediastinum and more frequently on the left side than the right. They are usually located anteriorly and deep to the SCM muscle. About half of ectopic thymuses are connected to the mediastinal thymus by a vestigial remnant or a solid cord. Ectopic thymic tissue is usually not palpable and typically is found incidentally on imaging studies performed for other reasons. Thus, the presentation age is variable, but because the thymic tissue regresses with age, most cases occur in young children. On CT and MRI, ectopic thymic tissue is homo-geneous and shows enhancement with contrast (see the figures). Needle biopsy reveals lymphocytes with abun-dance of T cells, which may be misinterpreted as T-cell lymphoma if the possibility of ectopic thymus is not considered.

Notes

Page 21: Head and Neck Imaging - Case Review Series - Challenge

325

C A S E 1 6 0

History: Hoarseness

1. Which of the following conditions are possible causes of this lesion? (Choose all that apply.) A. Osteoma B. Chondrosarcoma C. Misplaced vocal cord implant D. Venous vascular malformation E. Squamous cell carcinoma

2. Which of the following interventions is not used to treat vocal cord paralysis? A. Medialization laryngoplasty B. Collagen injection for medialization C. Gore-Tex/Silastic implantation D. Thyroplasty plastic prosthesis E. Botulinum injection

3. Which of the following represents the percentage of chondrosarcomas of the larynx that affect the cricoid cartilage? A. 0–20% B. 21–40% C. 41–60% D. 61–80% E. >80%

4. Which of the following sites is the classic location of a hemangioma of the larynx? A. Pharynx B. Supraglottis C. Glottis D. Subglottis E. Trachea

Page 22: Head and Neck Imaging - Case Review Series - Challenge

326

A N S W E R S

C A S E 1 6 0

Venous Malformation of the Larynx1. B, C, and D

2. E

3. D

4. D

ReferencesBent JP. Airway hemangiomas: contemporary manage-

ment. Lymphat Res Biol. 2003;1(4):331–335.Thompson LD, Gannon FH. Chondrosarcoma of the

larynx: a clinicopathologic study of 111 cases with a review of the literature. Am J Surg Pathol. 2002 Jul;26(7):836–851.

Cross-ReferenceNeuroradiology: THE REQUISITES, 3rd ed, pp 491–

492.

CommentsThis was a venous vascular malformation of the supra-glottis with a small phlebolith much better seen on the CT scan, which was maliciously not provided to the reader.

The major differential diagnosis is a chondroid lesion of the larynx. These are usually chondrosarcomas and favor men over women by a 3:1 ratio. Mean age is 61 years, and the clinical presentation is overwhelmingly a voice change. Around 77% affect the cricoid cartilage, 19% affect the thyroid cartilage, and less than 3% affect the arytenoid or epiglottis. Because they are low-grade lesions in more than 78%, they are treated conserva-tively, saving total laryngectomy for the last resort while trying to maintain a patent airway as the main goal. Mean survival is 7.5 years. Benign chondromas of the cartilage often coexist with chondrosarcomas, and one report noted that 60% of chondrosarcomas were superimposed on a pre-existing benign chondroma and may be induced by ischemia of the chondroma. Recurrence rates of chondrosarcomas are 30% to 40% but this is because of the nonaggressive surgical extirpation. Metastases occur in 2% to 10%.

The calcification in this lesion was much more indica-tive of a phlebolith of a venous vascular malformation than the typical popcorn stippled and coarse calcifica-tion of the cricoid cartilage seen in chondrosarcomas (see the figures).

Hemangiomas of the larynx are much more com-monly seen in the neonatal period and favor the subglot-tic region, where they may acutely narrow the airway. They are more common in girls and, like most heman-giomas, have a growth and involution phase. The inci-dence of concomitant cutaneous hemangiomas is about

50%. Treatment options include conservative watchful waiting, laser therapy, interferon, open resection, and steroids. Case reports of glottic and supraglottic heman-giomas abound, testifying to how rare an entity this is…or how rarely it is diagnosed.

Notes

Page 23: Head and Neck Imaging - Case Review Series - Challenge

327

C A S E 1 6 1

History: A 32-year-old man with a history of frequent epistaxis

1. Which of the following imaging features is most sug-gestive of a malignant process in this case? A. Hyperdense mass B. Extension outside the sinonasal cavity C. Focal bone destruction D. Thickening of the anterior wall of the sinus E. All of the above

2. Benign sinonasal mass lesions that may have an aggres-sive imaging appearance include all of the following EXCEPT: A. Angiectatic polyp B. Organized hematoma C. Antrochoanal polyp D. Juvenile angiofibroma E. Inverting papilloma

3. Sinonasal organized hematomas occur in the maxil-lary sinuses. True or false?

4. All of the following are histologic subtypes of inflam-matory sinonasal polyps EXCEPT: A. Edematous B. Glandular C. Fibrous D. Cystic E. Angiectatic F. All are subtypes of nasal polyps.

Page 24: Head and Neck Imaging - Case Review Series - Challenge

328

A N S W E R S

C A S E 1 6 1

Angiectatic Polyp and Maxillary Sinus Organized Hematoma1. C

2. C

3. True

4. F

ReferencesKim EY, Kim HJ, Chung SK, et al. Sinonasal organized

hematoma: CT and MR imaging findings. AJNR Am J Neuroradiol. 2008 Jun;29(6):1204–1208. Epub 10 Apr 2008.

Yfantis HG, Drachenberg CB, Gray W, Papadimitriou JC. Angiectatic nasal polyps that clinically simulate a malignant process: report of 2 cases and review of the literature. Arch Pathol Lab Med. 2000 Mar;124(3):406–410.

Cross-ReferenceNeuroradiology: THE REQUISITES, 3rd ed, p 433.

CommentAngiectatic sinonasal polyp is a rarely reported entity that is histologically characterized by an extensive pro-liferation of ectatic blood vessels with perivascular pseu-doamyloid deposition and reactive stromal myofibroblasts showing reactive/degenerative atypia. Some of the angi-ectatic polyps demonstrate rapid and expansive growth with erosion of bone and may be clinically mistaken for a malignant process. On imaging, a mass centered in the maxillary sinus with features suggesting an aggressive neoplasm, including bone destruction and extension outside the sinonasal cavity, are seen (see the figures). MRI shows blood by-products with varying ages, which may be a clue to the correct diagnosis. We believe angio-ectatic sinonasal polyps are the underlying lesions in at least some of the reported cases of organized hematoma of the sinuses.

Imaging findings that suggest a malignant sinonasal process include large mass, extension outside the sino-nasal cavity, and infiltration rather than displacement of the surrounding structures, although most benign masses can attain large sizes and extend outside the sinuses. The type of bone erosion present in association with a soft-tissue mass in the sinonasal cavity often provides insight as to the nature of the mass; slow-growing masses thin and remodel the bone, whereas fast-growing lesions focally destroy the bone before it gets a chance to adapt and remodel. Thickening of the sinus walls indicates a chronic and usually inflammatory process. Histologically benign lesions that often demonstrate aggressive imaging features include inverting papillomas, juvenile angio-

fibromas, and cellular schwannomas. Organized hema-toma of the maxillary sinus is a rarely reported entity that typically mimics a malignant lesion because of bone destruction and extension beyond the sinus. Angioec-tatic polyps, although very rare, act like malignant tumors as well. Small malignant tumors, on the other hand, may not appear as aggressive on imaging.

Notes

Page 25: Head and Neck Imaging - Case Review Series - Challenge

329

C A S E 1 6 2

History: Neck pain, headaches

1. Which of the following disorders should the differen-tial diagnosis for this case include? (Choose all that apply.) A. Nasopharyngeal carcinoma B. Juvenile nasopharyngeal angiofibroma C. Squamous cell carcinoma D. Lymphoma E. Chordoma

2. Which of the following characteristics allow one to differentiate between retropharyngeal lesions and prevertebral lesions? A. The mucosa is displaced anteriorly with one but

not the other. B. The parapharyngeal fat is displayed anteriorly

with one but not the other. C. One displaces the longus colli anteriorly, the other

posteriorly. D. One displaces the carotid sheath anteriorly and

the other does not. E. None of the above

3. In which direction do most ecchordosis physaliphora grow from the clivus? A. Anteriorly B. Posteriorly C. Laterally D. Medially E. All of the above

4. What percentage of chordomas arises from the cervi-cal spine? A. 1–20% B. 21–40% C. 41–60% D. 61–80% E. None of the above

Page 26: Head and Neck Imaging - Case Review Series - Challenge

330

A N S W E R S

C A S E 1 6 2

Chordoma Presenting as a Prevertebral Mass1. A, D, and E

2. C

3. B

4. A

ReferenceSciubba DM, Chi JH, Rhines LD, Gokaslan ZL. Chordoma

of the spinal column. Neurosurg Clin North Am. 2008 Jan;19(1):5–15.

Cross-ReferenceNeuroradiology: THE REQUISITES, 3rd ed, pp 383, 502.

CommentThe cervical spine is a less common site of origin for chordomas compared to the clivus and the sacrococcy-geal region, which are nearly equally represented by about 45% of cases. Nonetheless the appearance, being very bright on a T2W image and having enhancement and heterogeneous matrix, is typical of the tumor in all locations.

Just looking at the soft-tissue windows of the neck CT, few neuroradiologists would have correctly surmised the spinal origin to this case. You really cannot discern the longus colli muscles to determine in which direction they are displaced. This is the critical finding (best dem-onstrated on the T2W image), which will suggest that the lesion is from the perivertebral space (displacing the longus musculature anteriorly) or from the mucosal or retropharyngeal space (displacing the longus muscula-ture posteriorly) (see the figures). One of the lesions that could have confused the reader would have been an inflammation of the longus musculature itself, so-called calcific tendinitis of the longus musculature. This could even simulate the matrix of a chordoma (missing here). However, the bony nature of the lesion is demonsrated to the observant reviewer on the bone window CT and certainly by the time one reviews the T2W image. Ret-ropharyngeal inflammation could also have entered the differential diagnosis if the lesion was misplaced into the retropharyngeal space. In this location, necrotizing ade-nitis is the most common inflammatory condition.

Notocord remnants can be found throughout the spine and are the progenitors of the nucleus pulposus. There is a benign entity known as ecchordosis phys-aliphora, which presents as a bony lesion emanating from the posterior wall of the clivus in 2% of the popula-tion. As such it may simulate a chordoma or a calcified

meningioma. It should be asymptomatic, however, and in contrast to the other entities, it often does not enhance. It may attach to the clivus by a small pedicle.

Notes

Page 27: Head and Neck Imaging - Case Review Series - Challenge

331

C A S E 1 6 3

History: A 5-year-old boy who failed the school hearing screening test

1. Which type of hearing loss do you expect this boy to have? A. Conductive hearing loss B. Sensorineural hearing loss C. Mixed

2. Which of the following represents the most common reason for conductive hearing loss in children? A. Ossicular fusion B. Oval window agenesis C. Otitis media and its sequelae D. Cochlear malformation E. Vestibular malformation

3. Which of the following structures is of greatest impor-tance in oval window surgery? A. Facial nerve B. Stapes footplate C. Vestibule D. Stapedius muscle E. Incudostapedial joint

4. The normal facial nerve travels underneath the lateral semicircular canal just above the oval window. Dehis-cence of the facial canal may result in protrusion of the facial nerve into the oval window. True or false?

Page 28: Head and Neck Imaging - Case Review Series - Challenge

332

A N S W E R S

C A S E 1 6 3

Oval Window Agenesis and Aberrant Facial Nerve1. A

2. C

3. A

4. True

Referencede Alarcon A, Jahrsdoerfer RA, Kesser BW. Congenital

absence of the oval window: diagnosis, surgery, and audiometric outcomes. Otol Neurotol. 2008 Jan;29(1):23–28.

Cross-ReferenceNeuroradiology: THE REQUISITES, 3rd ed, p 390.

CommentAgenesis of the oval window is an uncommon cause of congenital conductive hearing loss. In this anomaly, the oval window is completely covered with bone and the stapes footplate fails to form. The crura of the stapes may be normal or malformed. The facial nerve may not be in its normal anatomic location, which presents a signifi-cant challenge for the surgical correction of this problem, which is usually accomplished by drilling the oval window. It is the responsibility of the radiologist to determine the course of the facial nerve on CT scans before all surgical procedures involving the otic capsule. When an anomaly of the otic capsule is present, the likelihood of an aberrant facial nerve increases dramati-cally. Identification of such an aberrant course is crucial for surgical eligibility as well as the type of surgical approach. The most common abnormality in the course of the facial nerve is its medial and anterior displace-ment, as seen in this case (see the figures). In this sce-nario, the facial nerve overlies the oval window and makes its second turn just posterior to the oval window rather than in the facial recess, which is more lateral and posterior.

Notes

Page 29: Head and Neck Imaging - Case Review Series - Challenge

333

C A S E 1 6 4

History: Left jaw pain

1. Which of the following disorders are the most likely diagnoses? (Choose all that apply.) A. Dentigerous cyst B. Radicular cyst C. Odontogenic keratocyst D. Complex odontoma E. Ameloblastoma

2. Which of the following conditions is associated with odontogenic keratocysts (keratocystic odontogenic tumor)? A. Gorham’s syndrome B. Gotham’s syndrome C. Blue rubber bleb nevus syndrome D. Gorlin’s syndrome E. Gardner’s syndrome

3. How can ADC maps differentiate keratocystic odon-togenic tumors (KOTs) from ameloblastomas? A. ADCs are higher in nonenhancing portions of

ameloblastomas than in KOTs. B. ADCs are lower in nonenhancing portions of ame-

loblastomas than in KOTs. C. ADCs are higher in enhancing portions of amelo-

blastomas than in KOTs. D. None of the above E. All of the above

4. Ameloblastomas have more solid and more enhancing components than KOT. True or false?

Page 30: Head and Neck Imaging - Case Review Series - Challenge

334

A N S W E R S

C A S E 1 6 4

Ameloblastoma1. E

2. D

3. A

4. True

ReferenceSumi M, Ichikawa Y, Katayama I, et al. Diffusion-weighted

MR imaging of ameloblastomas and keratocystic odon-togenic tumors: differentiation by apparent diffusion coefficients of cystic lesions. AJNR Am J Neuroradiol. 2008 Nov;29(10):1897–1901. Epub 21 Aug 2008.

Cross-ReferenceNeuroradiology: THE REQUISITES, 3rd ed, p 456.

CommentAmeloblastomas usually affect the mandible in the molar region. They have both solid and cystic components and may also have solid enhancing areas, on MRI in particular (see the figures). The nonenhancing components have higher ADC values than those of KOT due to the higher levels of desquamated keratin in the KOT, which raises the cystic viscosity and lowers the ADC value. If the jaw lesion is multicystic with solid components, bet on ame-loblastoma. If unilocular and along the long axis of the mandible, favor KOT.

A number of dental lesions are associated with syn-dromes. KOT is associated with Gorlin’s basal cell nevus syndrome. Osteomas, supernumerary teeth, and seba-ceous cysts are associated with Gardner’s syndrome. Cherubism is associated with Noonan’s syndrome. SAPHO syndrome may have periosteal reactions along the mandible. Down syndrome has multiple dental caries. Fibrous dysplasia occurs with McCune-Albright syndrome.

Odontomas are the most common dense lesions of the jawbones, accounting for more than two thirds of those reported. Compound ones affect the maxilla and show cystic areas with small toothlike radiodensities, whereas complex odontomas affect the mandible and are more solidly dense and calcified.

Ameloblastoma is the 2nd most common odontogenic tumor. 80% affect the mandible, and 80% are multilocu-lar. 80% of the multilocular varieties recur.

Notes

Page 31: Head and Neck Imaging - Case Review Series - Challenge

335

C A S E 1 6 5

History: 37-year-old man with sensorineural hearing loss (see the figure)

1. The differential diagnosis of a focal lucency in the otic capsule includes all of the following EXCEPT: A. Osteopetrosis B. Otosclerosis C. Osteogenesis imperfecta (OI) D. Normal pediatric ear E. Syphilis

2. Which of the following represents the most common reason for radiolucent defects in the otic capsule? A. Otosclerosis B. Osteogenesis imperfecta C. Paget’s disease D. Fibrous dysplasia E. Metastasis

3. Which of the following sites is the characteristic loca-tion of otic capsule demineralization in otosclerosis? A. Around the apical turn of the cochlea B. Around the basilar turn of the cochlea C. Fissula ante fenestram D. Fissula retro fenestram E. Cochlear aqueduct

4. The classic presentation of otosclerosis is slowly pro-gressing bilateral sensorineural hearing loss. True or false?

Page 32: Head and Neck Imaging - Case Review Series - Challenge

336

A N S W E R S

C A S E 1 6 5

Osteogenesis Imperfecta1. A

2. A

3. C

4. False

ReferenceAlkadhi H, Rissmann D, Kollias SS. Osteogenesis imper-

fecta of the temporal bone: CT and MR imaging in Van der Hoeve-de Kleyn syndrome. AJNR Am J Neurora-diol. 2004 Jun–Jul;25(6):1106–1109.

Cross-ReferenceNeuroradiology: THE REQUISITES, 3rd ed, pp 408–

410.

CommentThe otic capsule, the bony structure surrounding the membranous labyrinth, is the hardest bone in the body and normally appears uniformly hyperdense on CT in adults. Demineralization of the otic capsule presents as areas of decreased attenuation on CT; these can be focal or confluent. The most common reason for otic capsule demineralization is otosclerosis (otospongiosis), a bone dystrophy limited to the otic capsule. Otosclerosis is a frequent cause of conductive hearing loss in adults. The bony changes typically start in the fissula ante fenestram region, which is immediately anterior to the oval window and stapes footplate, and result in fixation of the stapes (fenestral otosclerosis). The fissula ante fenestram is histologically different from the rest of the otic capsule in that it contains fibrous and cartilaginous tissue. In advanced cases the entire otic capsule is involved (ret-rofenestral otosclerosis) with development of sensori-neural hearing loss on top of conductive hearing loss. OI is a hereditary connective tissue disorder that presents in clinically distinct forms. The OI tarda subtype presents later in life and can involve the temporal bone in a similar fashion as retrofenestral otosclerosis (see the figure). Differentiation of retrofenestral otosclerosis from OI on CT is only possible if the fissula ante fenestram is spared in the presence of otic capsule demineralization, as seen in this case. Because of systemic manifestations, OI is usually diagnosed before hearing loss develops. Other causes of otic capsule demineralization include Paget’s disease, syphilis, and some bone dysplasias. Osteopetro-sis can involve the temporal bone but it is associated with sclerosis rather than demineralization. Small areas of lucency in the otic capsules of young children are seen frequently and should not be confused with a disease process.

Notes

Page 33: Head and Neck Imaging - Case Review Series - Challenge

337

C A S E 1 6 6

History: Bleeding from the throat in a teenager

1. Which of the following cancers would be included in the differential diagnosis of this mass given the history? (Choose all that apply.) A. Nasopharyngeal carcinoma B. Juvenile nasopharyngeal angiofibroma (JNA) C. Squamous cell carcinoma D. Lymphoma

2. Which of the following is a risk factor for nasopha-ryngeal carcinoma? A. African American B. EBV exposure C. Alcohol D. Smoking E. All of the above

3. Which of the following offers the most precise description of the site of origin of this mass? A. Nasal cavity B. Nasopharynx C. Pterygopalatine fossa D. Nasopalatine canal E. Sphenopalatine foramen

4. Which of the following foramina is inferomedial to the foramen rotundum? A. Vidian canal B. Foramen ovale C. Foramen spinosum D. Optic canal E. None of the above

Page 34: Head and Neck Imaging - Case Review Series - Challenge

338

A N S W E R S

C A S E 1 6 6

Juvenile Nasopharyngeal Angiofibroma1. A, B, and D

2. B

3. E

4. A

ReferenceLloyd G, Howard D, Lund VJ, Savy L. Imaging for juvenile

angiofibroma. J Laryngol Otol. 2000 Sep;114(9):727–730.

Cross-ReferenceNeuroradiology: THE REQUISITES, 3rd ed, p 383.

CommentThis is not a typical growth pattern for JNA (which is why it is placed in the “hard” category). The typical JNA has more nasopharyngeal and nasal cavity growth, whereas this one seemed to turn north and head into the sphenoid sinus region (see the figures). The tumors origi-nate from the sphenopalatine foramen, the medial egress from the pterygopalatine fossa, but they may extend through the various exits from the pterygopalatine fossa. Therefore, it is critical to scan the vidian canal, foramen rotundum, pterygomaxillary fissure, inferior orbital fissure, and palatine foramina for tumor growth.

The JNA typically afflicts teenage boys; the classic history is nasal bleeding. There is clearly a genetic pre-disposition on the Y chromosome because the lesion is decidedly uncommon in females. Treatment may be by excision preceded by endovascular embolization to reduce blood loss. This is another example (like paragan-gliomas) in which the classic teaching is that you see flow voids in the lesion, but in our experience this is not so common. However, the lesion will enhance just as strongly as paraganglioma.

Notes

Page 35: Head and Neck Imaging - Case Review Series - Challenge

339

C A S E 1 6 7

History: A 53-year-old woman presents with a swollen, red left eye.

1. All of the following disorders are included in the dif-ferential diagnosis EXCEPT: A. Wegener’s granulomatosis B. Lymphoma C. Inflammatory pseudotumor D. Ruptured dermoid E. Xanthogranulomatous disease

2. All of the following are subtypes of adult orbital xan-thogranulomatous disease EXCEPT: A. Adult-onset xanthogranuloma B. Necrobiotic xanthogranuloma C. Erdheim-Chester disease D. Adult-onset asthma and periocular

xanthogranuloma E. Xanthogranulomatoid pseudogranuloma

3. Steroid-responsive lesions of the lacrimal gland include all of the following EXCEPT: A. Lymphoma B. Pseudotumor C. Sarcoidosis D. Acute dacryoadenitis E. Chronic dacryoadenitis

4. The histopathologic hallmarks of adult orbital xantho-granulomatous disease are foamy histiocytes and Touton giant cells. True or false?

Page 36: Head and Neck Imaging - Case Review Series - Challenge

340

A N S W E R S

C A S E 1 6 7

Adult Orbital Xanthogranulomatous Disease1. D

2. E

3. D

4. True

ReferenceGuo J, Wang J. Adult orbital xanthogranulomatous

disease: review of the literature. Arch Pathol Lab Med. 2009 Dec;133(12):1994–1997.

Cross-ReferenceNeuroradiology: THE REQUISITES, 3rd ed, p 353.

CommentAdult xanthogranulomatous disease involving the orbit is rare and encompasses a group of entities with varying manifestations that are included under non-Langerhans cell histiocytic disorders. Clinically they typically present as swelling of the eye, which usually responds to steroid treatment. This leads to a misdiagnosis of an inflamma-tory process such as pseudotumor. Involvement of other organ systems is frequent. Diagnosis requires biopsy, although the frequent presence of lymphatic prolifera-tion presents a significant difficulty for pathologists in differentiating this entity from lymphoma or other lym-phoproliferative disorders of the orbit. More character-istic histologic features include foamy histiocytes, Touton giant cells, and varying degrees of fibrosis. On imaging studies it presents as a nonencapsulated, solid soft-tissue mass in the lacrimal fossa with infiltration of the surrounding structures, including the periorbital fat, extraocular muscles, skin, and subcutaneous tissue. The imaging differential diagnosis primarily includes inflam-matory pseudotumor and lymphoma (see the figures). Acute or chronic infections of the lacrimal gland as well as a primary malignancy with infiltrating features may also be considered in the differential.

Notes

Page 37: Head and Neck Imaging - Case Review Series - Challenge

341

C A S E 1 6 8

History: 1-year-old with leukocoria

1. Which of the following should be included in the dif-ferential diagnosis of leukocoria? (Choose all that apply.) A. Retinoblastoma B. Persistent hyperplastic primary vitreous (PHPV) C. Coat’s disease D. Toxocariasis E. Retinopathy of prematurity (ROP)

2. Calcification within an intraocular mass is most common in which of the following disorders? A. Retinoblastoma B. PHPV C. Coat’s disease D. Toxocariasis E. ROP

3. Intraocular mass in a small globe (microphthalmia) is most commonly secondary to which of the following disorders? A. Retinoblastoma B. PHPV C. Coat’s disease D. Toxocariasis E. ROP

4. The ocular masslike tissue in Coat’s disease does not show enhancement. True or false?

Page 38: Head and Neck Imaging - Case Review Series - Challenge

342

A N S W E R S

C A S E 1 6 8

Persistent Hypertrophic Primary Vitreous1. A, B, C, D, and E

2. A

3. B

4. True

ReferenceEdward DP, Mafee MF, Garcia-Valenzuela E, Weiss RA.

Coats’ disease and persistent hyperplastic primary vit-reous. Role of MR imaging and CT. Radiol Clin North Am. 1998 Nov;36(6):1119–1131.

Cross-ReferenceNeuroradiology: THE REQUISITES, 3rd ed, p 322.

CommentThe normal retina gives a red reflection to incident light. Replacement of the normal red reflection by a white reflection is called leukocoria; it can be seen in a variety of conditions. The most common reason for leukocoria is congenital cataract, which is diagnosed clinically early in life and does not come to the attention of the radiolo-gist. Retinoblastoma is the most common reason for leu-kocoria in the imaging population, followed by PHPV, Toxocara infection, Coat’s disease, and ROP.

During embryonic life, the primary vitreous is formed first and is gradually replaced by the secondary vitreous, which develops into the vitreous body. PHPV is caused by lack of regression and hyperplasia of fibrovascular tissue derived from the primary vitreous and its hyaloid arterial supply. PHPV is usually sporadic but can be found in association with Walker-Warburg syndrome, Norrie disease, or other ocular and systemic disorders. The typical imaging feature of PHPV is a triangular-shaped retrolental soft-tissue mass that extends to the optic nerve head in a linear fashion, which is the persis-tent hyaloid/Cloquet’s canal (see the figures). Retinal detachment is nearly always present with varying degrees of vitreal hemorrhage. The lens is frequently abnormal in shape and signal. Doppler US may show flow in the tubular-shaped intravitreal tissue. PHPV is often unilat-eral and occurs in microphthalmic globes, which is the most important feature that helps differentiate PHPV from retinoblastoma, which almost always occurs in normal-sized or enlarged globes. Calcification is a very common feature of retinoblastoma, whereas it is extremely rare in PHPV. CT is the imaging study of choice for leukocoria because it is much superior to MRI in detection of calcification.

In a child with leukocoria and an intraocular mass, the following features are helpful in the differential diagno-sis: calcification is very suspicious for retinoblastoma,

but Coat’s disease may also show calcification, albeit much less frequently. Toxocariasis and ROP very rarely present with calcification. In Coat’s disease the involved eye is usually slightly smaller than the normal eye. ROP occurs in premature babies; a history of term birth practi-cally eliminates this possibility. A history of prior eye infection is usually present with toxocariasis. The average age at presentation is 6 years for Coat’s disease and 18 months for retinoblastoma. Unlike retinoblastoma, the intraocular “mass” of Coat’s disease does not show enhancement on MRI.

Notes

Page 39: Head and Neck Imaging - Case Review Series - Challenge

343

C A S E 1 6 9

History: A 49-year-old woman with proptosis

1. Which of the following disorders should be included in the differential diagnosis? (Choose all that apply.) A. Meningioma B. Fibrous dysplasia C. Osteoblastic metastasis D. Osteosarcoma

2. Which of the following represents the most common site of calvarial (intraosseous) meningiomas? A. Frontal bone B. Parietal bone C. Temporal bone D. Sphenoid bone

3. All of the following locations are possible sites of extradural (ectopic) meningiomas EXCEPT: A. Paranasal sinuses B. Middle ear C. Lungs D. Stomach E. Parotid gland

4. Intraosseous meningiomas arise from sites of cranial suture or prior fractures. True or false?

Page 40: Head and Neck Imaging - Case Review Series - Challenge

344

A N S W E R S

C A S E 1 6 9

Intraosseous Meningioma1. A, B, C, and D

2. D

3. D

4. False

ReferenceElder JB, Atkinson R, Zee CS, Chen TC. Primary intraos-

seous meningioma. Neurosurg Focus. 2007;23(4):E13.

Cross-ReferenceNeuroradiology: THE REQUISITES, 3rd ed, p 352.

CommentMost meningiomas are intracranial, and when they approximate a portion of the skull, they often cause hyperostosis, which is a reactive sclerotic thickening of the bone. Infrequently they will cause lytic bone destruc-tion and extend outside the skull. About 2% of meningio-mas arise from outside the intracranial compartment and are named primary extradural meningiomas; these most commonly occur in the cranium, which is different from the reactive bony changes described above. The origin of primary intraosseous meningioma is controver-sial; they may arise from arachnoid cells trapped in the bone during embryologic life or from multipotent mes-enchymal cells. They typically occur in the sphenoid wing and less frequently in the frontal bone. They appear as very sclerotic expansile bone lesions on CT with often spiculated margins (see the figures). This appearance alone is helpful to differentiate intraosseous meningioma from fibrous dysplasia, which is less sclerotic and has a ground-glass density. Intraosseous meningioma may completely lack an extraosseous component but almost always demonstrates some enhancement of the adjacent dura on MRI. The soft-tissue component, when present, may grow into the orbit or intracranial compartment. Some intraosseous meningiomas will have a less sclerotic or even lytic appearance, in which case fibrous dysplasia and metastasis should be considered in the differential diagnosis. Presence of dural enhancement adjacent to the lesion helps differentiate intraosseous meningioma from fibrous dysplasia, which are the two most frequent sclerotic and expansile lesions of the sphenoid wing. The MRI signal of intraosseous meningioma is variable; very sclerotic lesions show dark signal on both T1W and T2W images and no enhancement, but some will have enhancing components. Osteoblastic metastases and bone-producing tumors such as osteosarcomas can be considered in the differential diagnosis, although

these are rare and clinical features are often helpful in differentiating these from intraosseous meningioma, which has a more benign and chronic course.

Notes

Page 41: Head and Neck Imaging - Case Review Series - Challenge

345

C A S E 1 7 0

History: Left neck mass

1. Given all patients who have a neck mass, which of the following should be included in the top three differential diagnoses? (Choose all that apply.) A. Reactive node B. Neoplastic node C. Thyroid mass D. Thyroglossal duct cyst E. Osteophyte

2. A thoracic duct lymphocele presents more commonly as a complication of surgery than de novo. True or false?

3. All of the following statements about thoracic duct lymphoceles are true EXCEPT: A. They are more commonly left-sided. B. They may be bright on T1W images. C. They are usually lateral to the jugular vein. D. They can result in a hemothorax. E. They can result in a chylothorax.

4. What of the following characteristics of a lymphocele of the thoracic duct distinguishes it from a node? A. Absence of enhancement B. Location C. Left-sidedness D. History of neck dissection E. All of the above

Page 42: Head and Neck Imaging - Case Review Series - Challenge

346

A N S W E R S

C A S E 1 7 0

Thoracic Duct Lymphocele1. A and B

2. True

3. D

4. A

ReferenceNouwen J, Hans S, Halimi P, Laccourreye O. Lymphocele

after neck dissection. Ann Otol Rhinol Laryngol. 2004 (Jan);113(1):39–42.

Cross-ReferenceNeuroradiology: THE REQUISITES, 3rd ed, p 511.

CommentThoracic duct lymphocele may occur de novo, but it is our experience that it is more commonly an incidental finding after a neck dissection (reported in 1–6%) or blunt trauma. The thoracic duct may be injured because the dissection occurs near the junction of the jugular vein and subclavian vein. In some cases this may appear as a tiny cyst in this typical location, probably dismissed as a small lymph node unless contrast is administered (see the figure). In that case, whether on MRI or CT one can see the cystic nature of the lesion and its likely origin.

The lesion may be bright on precontrast MRI due to high protein lymph fluid or fatty chylous material. It is always bright on T2W imaging. Aspiration finds the typical milky white chyle. The lesion is seen far more commonly in the left neck because that is where the main thoracic duct drains, but rarely it can be seen along lymph channels in the right neck. The lesion displaces the carotid sheath structures medially and is typically seen in the supraclavicular fossa. The lesion may expand with Valsalva maneuvers.

Treatment proposed is closed drainage, dietary restric-tion of fatty substances, and pressure dressings in cases where there is active gross leakage or chylothorax. Oth-erwise, in those incidental cases, it is best to leave the lesion alone due to its friable wall.

Notes

Page 43: Head and Neck Imaging - Case Review Series - Challenge

347

C A S E 1 7 1

History: A 42-year-old presents with recent-onset vertical double vision.

1. Which of the following findings are correct? (Choose all that apply.) A. Right superior oblique muscle is hypertrophic. B. Left superior oblique muscle is atrophic. C. Tendon of the right superior oblique muscle is

hypertrophic. D. Tendon of the left superior oblique muscle is

atrophic. E. Left trochlea is disrupted.

2. All of the following disorders should be included in the differential diagnosis EXCEPT: A. Orbital pseudotumor B. Brown syndrome C. White syndrome D. Lymphoma E. Leukemia

3. All of the following are causes of isolated superior oblique muscle enlargement EXCEPT: A. Thyroid orbitopathy B. Pseudotumor C. Lymphoma D. Leukemia

4. The most common cause of isolated trochlear nerve palsy is ischemia. True or false?

Page 44: Head and Neck Imaging - Case Review Series - Challenge

348

A N S W E R S

C A S E 1 7 1

Brown Syndrome1. A and C

2. C

3. A

4. False

ReferenceCurrie S, Goddard T. MR imaging features of acquired

Brown syndrome. AJNR Am J Neuroradiol. 2009 Oct;30(9):1778–1179. Epub 15 Apr 2009.

Cross-ReferenceNeuroradiology: THE REQUISITES, 3rd ed, p 342.

CommentBrown syndrome is characterized by the inability to gaze upward while adducting the eye, which occurs second-ary to an inflammation of the superior oblique tendon sheath complex and the trochlea itself where the tendon makes a right angle turn to attach to the globe. A pedi-atric form of the Brown syndrome also exists and is associated with a congenitally short superior oblique muscle tendon. The etiology of acquired Brown syn-drome is variable; it can be seen in association with systemic or local inflammatory processes such as rheu-matoid arthritis, SLE, sinusitis, scleritis, and trauma. The proximal tendon of the superior oblique muscle origi-nates from the lesser wing of the sphenoid. Its distal tendon loops through the trochlea (a fibrocartilaginous pulley attached to the medial orbital wall) before insert-ing into the sclera. In Brown syndrome, the inflamma-tory process primarily involves the distal tendon, but the body of the superior oblique muscle may enlarge as well. In some cases the abnormality is isolated to the trochlea. CT shows characteristic thickening of the muscle tendon and sheath, although MRI is more sensitive to inflamma-tory signal changes (see the figures). Treatment consists of anti-inflammatory drugs.

Notes

Page 45: Head and Neck Imaging - Case Review Series - Challenge

349

C A S E 1 7 2

History: Headaches

1. Which of the following disorders is the most likely diagnosis? A. Osteoradionecrosis B. Benign resorption of condyles C. Degenerative disease D. Gout E. Pseudogout

2. All of the following are associated with avascular necrosis of the mandibular condyle EXCEPT: A. Teflon-Proplast implant B. Caisson disease C. Sickle cell disease D. Trauma/fracture E. None of the above

3. Which of the following disorders is associated with mandibular condylar resorption? A. Trauma B. Osteoarthritis C. Avascular necrosis D. Collagen vascular disease E. All of the above

4. Cheerleaders syndrome is another name for idio-pathic resorption of the mandibular condyles because it occurs most commonly in teenage girls who par-ticipate in sports. True or false?

Page 46: Head and Neck Imaging - Case Review Series - Challenge

350

A N S W E R S

C A S E 1 7 2

Resorption of Mandibular Condyles1. B

2. E

3. E

4. True

ReferenceWolford LM. Idiopathic condylar resorption of the tem-

poromandibular joint in teenage girls (cheerleaders syndrome). Proc (Bayl Univ Med Cent). 2001 July;14(3):246–252.

Cross-ReferenceNeuroradiology: THE REQUISITES, 3rd ed, p 488.

CommentCheerleaders syndrome refers to idiopathic resorption of mandibular condyles, as seen in this case (see the figures). The entity is thought to be due to repetitive subclinical trauma to the jaw during sports activities. It affects girls more than boys by a 9:1 ratio, usually in the teenage years. It is distinctly uncommon after age 20. The dif-ferential diagnosis for mandibular resorption includes the entities listed in question 3 (trauma, osteoarthritis, avascular necrosis, collagen vascular disease, including rheumatoid arthritis, lupus, ankylosing spondylitis, scleroderma, and psoriatic arthritis). The idiopathic form shows a loss of the vertical height of the mandibular condyle and results in malocclusion and strain on the meniscus, leading to pain.

Because of the demographics involved, most oral sur-geons believe the entity is associated with estrogen hormone effects, similar to TMJ meniscal abnormalities. The same hormones that may produce ligamentous laxity of the retrodiskal tissues may lead to resorption of the bone and shrinkage.

Conservative treatment, splinting, is often ineffective. Surgical treatment is directed at removing hyperplastic synovial tissue, repositioning the meniscus if needed, and orthognathic correction of mandibular malposition to improve the bite.

Notes

Page 47: Head and Neck Imaging - Case Review Series - Challenge

351

C A S E 1 7 3

History: Newborn with respiratory distress

1. Which of the following is the most common nasal anomaly causing respiratory distress in newborns? A. Choanal atresia B. Nasal pyriform aperture stenosis C. Midnasal stenosis D. Cleft palate

2. The nasal pyriform aperture is considered stenosed when it measures less than: A. 14 mm B. 12 mm C. 10 mm D. 8 mm E. 6 mm

3. In choanal atresia, in which of the following locations does the bony fusion occur? A. Between the vomer and the pterygoid process B. Between the vomer and the maxilla C. Between the nasal septum and the maxilla D. Between the perpendicular plate of the ethmoid

and the pterygoid process

4. Central single maxillary tooth predicts the presence of midline malformations in the face and brain. True or false?

Page 48: Head and Neck Imaging - Case Review Series - Challenge

352

A N S W E R S

C A S E 1 7 3

Nasal Pyriform Aperture Stenosis1. A

2. C

3. A

4. True

ReferenceBelden CJ, Mancuso AA, Schmalfuss IM. CT features

of congenital nasal piriform aperture stenosis: initial experience. Radiology. (1999);213:495–501.

Cross-ReferenceNeuroradiology: THE REQUISITES, 3rd ed, p 421.

CommentNasal obstruction in the newborn is a potentially life-threatening condition because neonates are obligate nasal breathers. Nasal pyriform aperture stenosis (NPAS) is a rare cause of nasal obstruction in the infant. The pyriform aperture is the narrowest portion of the bony nasal airway. NPAS results from bony overgrowth of the nasal process of the maxilla. The distance between the two nasal processes is 12 mm on average in normal infants. This does not take into account the narrowing of the airway by the mucosa. A measurement below 10 mm indicates NPAS (it measures <6 mm in our case). The clinical presentation of NPAS is the same as choanal atresia, which is much more common than NPAS. When a 6F catheter cannot be passed through the nasal cavity, NPAS is suspected.

NPAS may be associated with other midline anoma-lies, including holoprosencephaly, pituitary deficiency, and the presence of a single central maxillary incisor tooth, as seen in this case (see the figures). A single central maxillary incisor is an anomalous tooth that occurs precisely in the midline of the maxillary arch. It is associated with nasal malformations, including choanal atresia and NPAS, cleft lip or palate, holoprosencephaly, hypopituitarism, and hypotelorism.

Nasal choanal atresia is the most common nasal devel-opmental anomaly that results in respiratory distress. In choanal atresia, the vomer (posterior part of the nasal septum) is hypertrophic and fused to the pterygoid process. Choanal atresia can be membranous rather than bony. The air column between the nasal septum and ipsilateral pterygoid process is about 6 mm in normal infants.

Notes

Page 49: Head and Neck Imaging - Case Review Series - Challenge

353

C A S E 1 7 4

History: A 35-year-old man with conductive hearing loss and history of chronic ear infection

1. Which of the following is the most common cause of conductive hearing loss in adults? A. Trauma B. Chronic otomastoiditis C. Acute otomastoiditis D. Otosclerosis E. Vestibular schwannoma

2. Which of the following conditions is implied by cal-cification in a middle ear mass? A. Glomus tympanicum B. Facial schwannoma C. Middle ear adenoma D. Inflammatory tissue E. Cholesteatoma

3. Which of the following is the most common site of ossicular erosion in cholesteatomas? A. Malleus B. Incudomalleolar joint C. Incudostapedial joint D. Stapes crura E. Stapes footplate

4. All of the following CT findings would favor choles-teatoma over inflammatory change EXCEPT: A. Erosion of the scutum B. Opacification of the lateral epitympanic space C. Erosion of the lateral semicircular canal D. Thickening of the tympanic membrane E. Masslike soft tissue in a nondependent aspect of

the middle ear

Page 50: Head and Neck Imaging - Case Review Series - Challenge

354

A N S W E R S

C A S E 1 7 4

Tympanosclerosis1. B

2. D

3. C

4. D

ReferenceLemmerling MM, De Foer B, VandeVyver V, et al.

Imaging of the opacified middle ear. Eur J Radiol. 2008 Jun;66(3):363–371. Epub 12 Mar 2008.

Cross-ReferenceNeuroradiology: THE REQUISITES, 3rd ed, p 392.

CommentA soft-tissue attenuation filling the middle ear cavity as a CT finding (see the figures) is difficult to interpret without appropriate clinical input because CT’s ability to differentiate acute or chronic effusions, fibrotic tissue, cholesteatomas, and mass lesions is limited. In an acutely infected ear (“draining ear”) the primary concern is an abscess collection, whereas in the chronic setting (“dry ear”) a differentiation between inflammatory fibrotic tissue and cholesteatoma is the main problem. Recurrent infections in the middle ear lead to formation of a dense fibrotic tissue that can be located in any site in the middle ear but frequently involves the entire middle ear cavity. Hyalinization of the collagen tissue can be seen in histopathologic specimens. In advanced stages, new bone formation occurs. To describe this continuum, dif-ferent names, including chronic adhesive otitis media, tympanosclerosis, and fibro-osseous sclerosis, have been used. Clinically these patients present with conductive hearing loss (CHL) secondary to fixation of the ossicles by fibrotic tissue or in some cases frank erosion of the ossicles. Thickening of the tympanic membrane that has lost its elasticity is another contributing factor to CHL. Ossicular fixation by fibrotic tissue most frequently occurs at the oval window–stapes footplate region, although there is no imaging correlate for this. In a patient with a history of chronic otitis media, dry ear, and CHL radiologists should look for calcifications about the middle ear ossicles and within the soft tissue to establish the diagnosis of tympanosclerosis. Occasionally there is no visible soft tissue in the middle ear, in which case the diagnosis may escape the radiologist. The integ-rity of the ossicular chain is very important for treatment decisions. The integrity of the ossicles can be established even when they are surrounded by soft tissue. The pres-ence of ossicular erosion should raise the possibility of cholesteatoma, although chronic inflammation may erode the ossicles as well. The parts of the ossicular

chain most vulnerable to erosion are the long and len-ticular processes of the incus and the head of the stapes, although any part of the ossicular chain can be eroded by cholesteatoma and inflammation. It is important to understand that small pieces of cholesteatomas may be present within the inflammatory tissue; in fact, choles-teatomas almost always coexist with inflammatory tissue; the distinction of these with CT is not always possible. The presence of bone erosion in the tegmen tympani, scutum, mastoid, and otic capsule strongly suggests cho-lesteatomas. When the middle ear is not completely opacified, differentiation of cholesteatomas from inflam-mation is easier; a soft-tissue “mass” in a nondependent portion of the middle ear is suspicious for cholestea-toma, as in most cases of tympanosclerosis the depen-dent portions of the middle ear are opacified. A masslike appearance also strongly favors cholesteatomas. MRI allows better differentiation of inflammatory tissue from cholesteatomas. If there is no history of otomastoiditis, one has to consider primary tumors of the middle ear. The most common primary tumor of the middle ear is glomus tympanicum, followed by facial nerve schwannoma. Primary meningiomas of the middle ear and middle ear adenomas are rare benign tumors. Although malignant tumors of the temporal bone may present clinically as a middle ear mass (tip of the iceberg for the otoscopist), it is extremely rare for a malignant tumor to be limited to the middle ear on imaging studies.

Notes

Page 51: Head and Neck Imaging - Case Review Series - Challenge

355

C A S E 1 7 5

History: A 37-year-old with jaw pain

1. Which of the following is the most common mass lesion of the temporomandibular joint? A. Osteochondroma B. Synovial osteochondromatosis C. Pigmented villonodular synovitis D. Chondrosarcoma E. Synovial sarcoma

2. Imaging findings of synovial osteochondromatosis include all of the following EXCEPT: A. Widening of the joint space B. Erosion and remodeling of the mandibular condyle C. Intracranial extension D. Extension into the masticator space E. Dark T2 signal in the joint F. Intra-articular calcifications

3. In what percentage of cases of synovial osteochon-dromatosis are intra-articular calcified bodies seen? A. 10% B. 30% C. 50% D. 70% E. 90%

4. Which of the following findings can differentiate pigmented villonodular synovitis from synovial osteochondromatosis? A. Type of bone erosion B. T2 signal C. Enhancement pattern D. Anatomic extension E. Demographics

Page 52: Head and Neck Imaging - Case Review Series - Challenge

356

A N S W E R S

C A S E 1 7 5

Synovial Osteochondromatosis of the TMJ1. B

2. E

3. C

4. B

ReferenceMeng J, Guo C, Yi B, et al. Clinical and radiologic

findings of synovial chondromatosis affecting the temporomandibular joint. Oral Surg Oral Med Oral Pathol Oral Radiol Endod. 2010 Mar;109(3):441–448. Epub 22 Jan 2010.

Cross-ReferenceNeuroradiology: THE REQUISITES, 3rd ed, pp 488–

491.

CommentsSynovial osteochondromatosis (SO) is a benign, tumor-like joint disorder characterized by chondrometaplasia of the synovial membrane and loose cartilaginous bodies in the joint space, which may or may not show calcifica-tion. SO usually involves large-extremity joints and is very rare in the temporomandibular joint (TMJ). It pres-ents on CT and MRI as enlargement of the joint space and remodeling of the mandibular condyle and the glenoid fossa. Extension to the masticator space is common; cranial extension into the temporal bone and middle cranial fossa may occur. Smooth remodeling of the bone rather than aggressive destruction is secondary to slow growth of these lesions, which usually take several years before they come to clinical attention. Cal-cified loose bodies within the mass are characteristic but not always present. Hypertrophied synovium and cartilaginous bodies are hyperintense on T2W MRI (see the figures). Postcontrast images show heterogeneous nodular enhancement.

Differential diagnoses include pigmented villonodular synovitis and more aggressive tumors like chondrosar-coma and other sarcomas.

Notes

Page 53: Head and Neck Imaging - Case Review Series - Challenge

357

C A S E 1 7 6

History: Thoracic outlet syndrome on the right

1. Which of the following disorders is the best diagnosis for this finding? A. Lymph node B. Accessory trapezius muscle C. Accessory sternocleidomastoid muscle D. Nodular fasciitis E. None of the above

2. Which of the following symptoms may occur from this finding? A. Thoracic outlet syndrome B. Cervical radiculopathy C. Fever, night sweats D. Restriction of motion E. None of the above

3. Is an accessory trapezius muscle more likely to affect C8–T1 roots or C6–C7 roots? A. C6–C7 B. C8–T1 C. C6–C7 and C8–T1 equally D. None of the above

4. All of the following statements about nodular fasciitis are true EXCEPT: A. It is a non-neoplastic fibroproliferative disorder. B. It may arise in the skin, subcutaneous tissue, deep

fascia, or muscle as a painful mass. C. It may regress on its own or may require surgery. D. It is best detected with T2W imaging with fat

suppression. E. It usually does not enhance.

Page 54: Head and Neck Imaging - Case Review Series - Challenge

358

A N S W E R S

C A S E 1 7 6

Accessory Trapezius Muscle(Contributed by Bruce Schlakman, University of Mississippi, Jackson, MS)

1. B

2. A

3. A

4. E

ReferenceHug U, Burg D, Meyer VE. Cervical outlet syndrome due

to an accessory part of the trapezius muscle in the posterior triangle of the neck. J Hand Surg Br. 2000 Jun;25(3):311–313.

Cross-ReferenceNeuroradiology: THE REQUISITES, 3rd ed, p 503.

CommentAn accessory trapezius muscle is a well-described source of compression of the C6–C7 roots or the upper trunk of the brachial plexus. It is of muscular density and intensity; hence its dark signal on the T2W image shown (see the figures). It is manifested as a muscular slip leading from the occipital region to a tendinous insertion on the clavicle and functions like a fibrous band to com-press the plexus in the scalene triangle. Other muscular anomalies that can compress the plexus are derived from the levator scapulae muscle and accessory middle scalene muscles.

Nodular fasciitis would be brighter on T2W imaging and typically enhances avidly. It may occur at any layer of the soft tissues of the neck. After the upper extremity, the head and neck is the 2nd most common site for this fibroproliferative disorder, representing 13% to 20% of cases. There is a relatively high rate of recurrence if incompletely resected. The lesion is often associated with the sternocleidomastoid muscle and may be initi-ated by trauma or infection. It may represent a repair mechanism gone bad.

Notes

Page 55: Head and Neck Imaging - Case Review Series - Challenge

359

C A S E 1 7 7

History: A 47-year-old man has a right-sided neck pain over the carotid artery, which is tender to touch.

1. All of the following conditions should be included in the differential diagnosis EXCEPT: A. Vasculitis B. Carotid dissection C. Carotidynia D. Glomus tumor

2. Clinical features of carotidynia include all of the fol-lowing EXCEPT: A. Unilateral neck pain B. Pain aggravated by swallowing C. Tenderness over the carotid bifurcation D. Accessory nerve paralysis E. Rapid response to steroids

3. All of the following statements regarding carotidynia are true EXCEPT: A. Thickening of the carotid wall is circumferential. B. It is limited to the distal common carotid artery. C. Definitive diagnosis requires biopsy. D. No blood by-product is seen within the inflamed

tissue. E. No calcification is seen within the inflamed tissue.

4. Recurrence occurs in 40% of cases. True or false?

Page 56: Head and Neck Imaging - Case Review Series - Challenge

360

A N S W E R S

C A S E 1 7 7

Carotidynia1. D

2. D

3. C

4. False

ReferencesFarage L, Motta AC, Goldenberg D, et al. Idiopathic

inflammatory pseudotumor of the carotid sheath. Arq Neuropsiquiatr. 2007 Dec;65(4B):1241–1244.

Kuhn J, Harzheim A, Horz R, Bewermeyer H. MRI and ultrasonographic imaging of a patient with caroti-dynia. Cephalalgia. 2006;26:483–485.

Cross-ReferenceNeuroradiology: THE REQUISITES, 3rd ed, p 495.

CommentCarotidynia is a poorly characterized idiopathic pain syn-drome that involves the neck. Patients usually present with unilateral neck pain and tenderness that increases with palpation of the carotid artery. Ultrasound, CT, and MRI show circumferential thickening of the wall of the distal common carotid artery just before the bifurcation with minimal, if any, narrowing of the lumen (see the figures). It is a self-limited condition and usually resolves with short-term anti-inflammatory treatment. Resolution of wall thickening can be demonstrated with follow-up imaging. In a few cases, pathologic findings have been described and include inflammatory infiltration without granuloma formation and varying degrees of fibrosis, which account for diffuse wall thickening on imaging studies with decreased T1 signal and enhancement on postcontrast images. T2 signal may be high or low, depending on the degree of fibrosis present at the time of imaging. Vasculitis should be considered in the dif-ferential diagnosis; the fact that wall thickening is limited to the distal common carotid artery is helpful in this regard as vessel wall thickening in vasculitis is more extensive. Other mass lesions that occur in the carotid sheath, such as metastatic lymph nodes, glomus tumors, and schwannomas, are usually much larger and, when small, do not involve the vessel wall circumferentially. Because of neck pain and arterial wall thickening, dissec-tion of the carotid artery may be considered in the dif-ferential diagnosis despite unusual location; in dissection MRI usually shows characteristic bright T1 signal associ-ated with subintimal blood by-products.

Notes

Page 57: Head and Neck Imaging - Case Review Series - Challenge

361

C A S E 1 7 8

History: Palpable mass in the right neck

1. Which of the following represents possible etiologies for this mass? (Choose all that apply.) A. Pleomorphic adenoma B. Submandibular astrocytic tumor C. Lymph node D. Schwannoma E. Mucosal squamous cell carcinoma

2. All of the following are characteristic MR imaging features of a pleomorphic adenoma EXCEPT: A. Avid enhancement B. Bright on T2W image C. Bright on T1W image D. All of the above E. None of the above

3. If this was a schwannoma, from which cranial nerve would this be arising? A. Lingual nerve B. Hypoglossal nerve C. Vagus nerve D. Finnegan’s nerve E. Spinal accessory nerve

4. Which of the following cells would aspiration of a schwannoma most likely yield? A. Schwann cells B. Fibroblasts C. Macrophages D. Red blood cells E. Spindle cells

Page 58: Head and Neck Imaging - Case Review Series - Challenge

362

A N S W E R S

C A S E 1 7 8

Submandibular Schwannoma1. A, C, and D

2. C

3. B

4. E

ReferenceKarpati RL, Loevner LA, Cunning DM, et al. Synchronous

hypoglossal nerve and sympathetic nervous system plexus schwannomas. AJR Am J Roentgenol. 1998;171(6):1505–1507.

Cross-ReferenceNeuroradiology: THE REQUISITES, 3rd ed, pp 492, 496,

and 725.

CommentSchwannomas of the head and neck are usually derived from branches of the trigeminal nerve, the facial nerve, the vagus nerve, and spinal nerves emanating from the cervical nerve roots. In this case, an unusual source of a schwannoma, the hypoglossal nerve, was found to be the nerve supplying this mass. Schwannomas are very frustrating from the standpoint of fine-needle aspiration sampling. They usually yield a “spindle cell neoplasm” diagnosis on cytology, which can include such benign entities as a schwannoma and nodular fasciitis, but they may also be seen in fibrosarcomas, malignant fibrous histiocytomas, and spindle cell sarcomas. Because of this, a true histologic sample is most often recommended at the time of aspiration. As one might expect, schwanno-mas can be very painful to sample and using the larger needle and taking a biopsy-sized chunk out of the mass may be uncomfortable for the patient.

In this case the question that arose was whether the mass was actually an intrinsic submandibular gland tumor. By the time the MRI was performed, more than a year after the CT scan, the submandibular gland was not distinguishable, likely totally atrophic, so that pleo-morphic adenoma of the submandibular gland (by virtue of its bright signal on T2W imaging) could have been a consideration. Of benign tumors of the submandibular gland, pleomorphic and monomorphic adenomas pre-dominate; Warthin’s tumors are not seen outside the parotid gland typically. The most common malignancy of the submandibular gland is the adenoid cystic carci-noma, but the current lesion is very well-defined and shows no perineural spread.

The coronal CT scan clearly shows that the mass is actually above the submandibular gland, not intrinsic to it (see the figures). In that case one should consider a

schwannoma. Given its size, it is unlikely to represent a metastatic lymph node because one might expect central necrosis in that scenario.

Notes

Page 59: Head and Neck Imaging - Case Review Series - Challenge

363

C A S E 1 7 9

History: A 36-year-old man with hearing loss following trauma

1. Based on these images, which of the following com-plications of temporal bone trauma is more likely to occur in this patient? A. Conductive hearing loss (CHL) B. Sensorineural hearing loss (SNHL) C. Facial nerve paralysis D. CSF leakage E. Perilymphatic fistula

2. Which of the following types of traumatic ossicular discontinuity is most common? A. Incudomalleolar joint separation B. Incudostapedial joint separation C. Dislocation of the incus D. Dislocation of the malleoincudal complex E. Stapediovestibular dislocation

3. Which of the following fracture classification schemes has a better correlation with clinical findings? A. Longitudinal versus transverse B. Oblique C. Otic capsule-sparing versus otic capsule-violating D. Petrous versus nonpetrous

4. Which of the following is the most common imaging finding in post-traumatic sensorineural hearing loss? A. Basilar turn of the cochlea fracture B. Apical turn of the cochlea fracture C. Vestibular fracture D. Semicircular canal fracture E. None of the above

Page 60: Head and Neck Imaging - Case Review Series - Challenge

364

A N S W E R S

C A S E 1 7 9

Incudomalleolar Joint Separation1. A

2. B

3. C

4. E

ReferenceSaraiya PV, Aygun N. Temporal bone fractures. Emerg

Radiol. 2009 Jul;16(4):255–265. Epub 4 Nov 2008.

Cross-ReferenceNeuroradiology: THE REQUISITES, 3rd ed, p 414.

CommentHearing loss is a common complication of trauma. Often this is transient. Of the permanent hearing losses associ-ated with trauma, SNHL is the most common type, fol-lowed by mixed SNHL and CHL. The overwhelming majority of post-traumatic SNHL cases are secondary to contusion of the membranous labyrinth and hence have no imaging correlate. Isolated CHL makes up only 20% of traumatic hearing loss cases. The majority of CHL is secondary to laceration of the tympanic membrane or hematotympanum and recovers spontaneously. If a patient continues to have CHL after the resolution of hematotympanum or tympanic membrane injury, a careful search for ossicular discontinuity should ensue. There are five general types of ossicular discontinuities: incudomalleolar joint separation (see the figure), incudo-stapedial joint separation, dislocation of the incus, dislocation of the malleoincudal complex, and stapedio-vestibular dislocation. The incus has minimal ligament support; therefore, it is most prone to displacement. The most common types of ossicular discontinuity are incu-dostapedial and incudomalleolar joint separations. Stape-diovestibular dislocation is the least common type of ossicular injury. Incudomalleolar joint separation is seen as separation of the “ice cream scoop from the cone” on axial images. Lack of alignment of the lenticular process of the incus and the head of the stapes indicates incudo-stapedial separation. Fractures of the lenticular process of the incus and the crura of the stapes are difficult to identify unless they are displaced. In children, fracture of the stapes is more common than in adults. Complete or partial resorption of an ossicle may occur shortly after traumatic injury.

When describing fractures of the temporal bone we classify them into groups of otic capsule-sparing and otic capsule-violating fractures because this scheme is clini-cally more helpful than just describing the anatomic plane of the fracture. Otic capsule-violating fractures are always associated with SNHL, whereas otic capsule-

sparing fractures are more likely to cause CHL. Demon-stration of involvement of an individual structure is more important than assigning the fracture into a general category, however. With the currently available high-resolution CT, the course of the entire facial nerve, cochlea, vestibule, and semicircular canals, vestibular and cochlear aqueducts, IAC, the middle ear ossicles, the external canal, and carotid canal should be meticulously examined in every patient with temporal bone trauma.

Notes

Page 61: Head and Neck Imaging - Case Review Series - Challenge

365

C A S E 1 8 0

History: Jaw pain in a patient with acute myelogenous leukemia (AML)

1. Which of the following conditions are possible diag-noses for these findings? (Choose all that apply.) A. Dental infection in an immunocompromised host B. Intraosseous hematoma C. Pathologic fracture D. Chloroma of the mandible E. None of the above

2. What is a chloroma? A. Extramedullary deposits of leukemia B. Congenital rests C. A type of rhabdomyosarcoma D. Lymph nodes of leukemia E. None of the above

3. Myelodysplastic syndromes may evolve into AML. True or false?

4. Chloromas occur in which of the following percent-age of patients with leukemia? A. >75% B. 50–75% C. 10–49% D. <10%

Page 62: Head and Neck Imaging - Case Review Series - Challenge

366

A N S W E R S

C A S E 1 8 0

Jaw Chloroma1. A, C, and D

2. A

3. True

4. D

ReferenceFritz J, Vogel W, Bares R, Horger M. Radiologic spectrum

of extramedullary relapse of myelogenous leukemia in adults. AJR Am J Roentgenol. 2007 Jul;189(1):209–218.

Cross-ReferenceNeuroradiology: THE REQUISITES, 3rd ed, p 492.

CommentChloroma and granulocytic sarcoma are the terms used for bony or soft-tissue infiltrates of AML, which can in some cases (30%) precede the hematologic manifesta-tion of the disease. In the head and neck, the orbit, sinonasal cavity, and bones of the skull and face may be affected in about 3% of patients with myelogenous leu-kemia. Most are solitary lesions that have a greenish hue secondary to the myeloperoxidase concentration in the mass.

The prognosis of a patient with and without a chlo-roma does not vary, but the chloroma may portend an ensuing blastic crisis.

The lesion usually enhances and is bright on most pulse sequences (see the figures). It may be bright on T1W imaging but, when occurring with the bone marrow fat, will not seem so obviously bright on T1W imaging. The lesions are occasionally hemorrhagic and in the CNS may represent as dural/meningeal processes, subependy-mal masses more than parenchymal lesions. In the head and neck, the skin, lips, and gingiva are the locations most commonly described.

AML is the most common leukemia in adults and is more relentless and aggressive than CLL or ALL. It affects adults more than children, with an average age of 65 years at presentation. Males, smokers, patients exposed to radiation, and patients with myelodysplastic syn-dromes are at risk for developing AML. In those patients receiving treatments for myeloid leukemia, such as allo-geneic stem cell transplantation, donor lymphocyte infu-sion, and second transplantation, the rate of chloroma development increases severalfold.

Treatment is primarily with radiation.

Notes

Page 63: Head and Neck Imaging - Case Review Series - Challenge

367

C A S E 1 8 1

History: Newborn baby with a left-sided neck mass iden-tified on prenatal ultrasound

1. Which of the following is the most common solid tumor in the neck in the neonatal period? A. Teratoma B. Neuroblastoma C. Rhabdomyosarcoma D. Adenopathy E. Hemangioma

2. Which of the following is the most common cystic neck mass in the neonatal period? A. Thyroglossal duct cyst B. Branchial apparatus cysts C. Venous malformation/hemangioma D. Lymphangioma E. Thymic cyst

3. All of the following statements regarding congenital neck neuroblastomas are true EXCEPT: A. They are solitary mass lesions. B. They mostly occur posterior to the carotid artery. C. The primary is in the retroperitoneum. D. They have a favorable prognosis compared to

infant/toddler neuroblastomas.

4. Which of the following congenital neck masses is more likely to cause airway compromise during birth? A. Teratoma B. Lymphangioma C. Neuroblastoma D. Hemangioma E. Esophageal duplication cyst

Page 64: Head and Neck Imaging - Case Review Series - Challenge

368

A N S W E R S

C A S E 1 8 1

Neuroblastoma1. A

2. D

3. C

4. A

ReferenceAbramson SJ, Berdon WE, Ruzal-Shapiro C, et al. Cervical

neuroblastoma in eleven infants—a tumor with favor-able prognosis. Clinical and radiologic (US, CT, MRI) findings. Pediatr Radiol. 1993;23(4):253–257.

Cross-ReferenceNeuroradiology: THE REQUISITES, 3rd ed, p 511.

CommentCongenital mass lesions in the neck are often diagnosed on prenatal ultrasound or at birth. Cystic mass lesions are the most common congenital masses. Teratomas are bulky, heterogeneous mass lesions that can pose prob-lems with birth and airway maintenance. Neuroblasto-mas that originate in the neck arise from the sympathetic chain in the vicinity of the carotid artery, as demon-strated in this case (see the figures). They are typically posterior to the carotid artery. Horner’s syndrome is a common finding in primary neck neuroblastomas. Metastatic neuroblastoma in the head and neck can involve lymph nodes, calvarium, and orbits, causing bone destruction and spiculated periosteal reaction.

Of pediatric rhabdomyosarcomas, 40% affect the head and neck, often presenting in the subcutaneous soft tissues, nasal cavity, orbit, middle ear, paranasal sinuses, and nasopharynx. Associated bony destruction is common but does not occur in all patients. Confluent inflammatory or infectious cervical lymph nodes can appear masslike, be solid or necrotic, or have variable vascularity and no specific imaging characteristics.

Fibromatosis colli can clinically mimic true neoplasms and has characteristic ultrasonographic findings of focal thickening of the sternocleidomastoid muscle without associated extramuscular abnormality. Congenital hem-angiomas can also mimic other mass lesions given their highly infiltrative appearance. Macrocystic lymphatic malformation is typically a multilocular fluid-filled mass rather than a purely solid lesion. Microcystic lymphatic malformation contains multiple small cysts that can enhance, similar to a solid mass.

Congenital and Neonatal Neck MassesLymphatic malformationThyroglossal duct cystBranchial cleft cystHemangioma/vascular malformationThymic cystDermoid cystEctopic thymusTeratomaNeuroblastomaRhabdomyosarcomaAdenopathyFibromatosis colli

Notes

Page 65: Head and Neck Imaging - Case Review Series - Challenge

369

C A S E 1 8 2

History: Right toothache

1. Which of the following are the possible causes of the bone lesion in this patient given the history provided? (Choose all that apply.) A. Pathologic fracture due to steroid administration B. Radicular cyst C. Osteonecrosis from bisphosphonate administra-

tion D. Osteomyelitis due to immunocompromised state E. Myxoma

2. Which of the following statements regarding giant cell tumors of the maxilla is correct? A. They may arise from a reparative granuloma. B. They are associated with fluorinated water. C. They are more common in patients with CLL. D. They metastasize in 20% to 40% of cases. E. None of the above

3. All of the following statements regarding myxomas are correct EXCEPT: A. Maxillary myxomas occur more commonly than

mandibular ones. B. They are usually cystic. C. They are associated with tiny supernumerary

teeth. D. They are not associated with plasma cell

dyscrasias. E. All of the above

4. Which of the following percentages represents the rate of invasion of the maxillary sinus by an odonto-genic myxoma? A. 0–20% B. 21–40% C. 41–60% D. 61–80% E. >80%

Page 66: Head and Neck Imaging - Case Review Series - Challenge

370

A N S W E R S

C A S E 1 8 2

Myxoma of the Maxilla1. E

2. A

3. C

4. D

ReferenceLi TJ, Sun LS, Luo HY. Odontogenic myxoma: a clinico-

pathologic study of 25 cases. Arch Pathol Lab Med. 2006 Dec;130(12):1799–1806.

Cross-ReferenceNeuroradiology: THE REQUISITES, 3rd ed, pp 450–

451.

CommentOdontogenic myxomas are mesenchymal tumors that slightly favor the maxilla over the mandible and usually arise in the molar region. They represent about 5% of all odontogenic tumors. Myxomas are much less common than ameloblastomas and odontomas in the incidence of odontogenic tumors and are dwarfed as well by non-odontogenic neoplasms that affect the mandible and maxilla. They invade the maxillary sinus frequently. They are seen as scalloping lesions that are usually multilocu-lar with ill-defined borders when maxillary in origin (see the figures). Teeth may be displaced, but the roots of the teeth should not be resorbed. Treatment options include block/segmental resection and partial or total maxillec-tomy or mandibulectomy.

A growing series of pediatric cases has been reported, but this is usually a tumor of young adulthood. It may be an asymptomatic finding on a screening sinus CT. The differential diagnosis includes ameloblastoma, giant cell reparative granulomas, and giant cell tumors. It often has a bubbly shape to it with lesser enhancement than ame-loblastoma. It is very bright on T2W images.

Notes

Page 67: Head and Neck Imaging - Case Review Series - Challenge

371

C A S E 1 8 3

History: An 8-week-old with nasal obstruction since birth

1. Which of the following entities should be included in the differential diagnosis of an intranasal mass in an infant? (Choose all that apply.) A. Hemangioma B. Dacrocystocele C. Epidermoid cyst D. Encephalocele E. Mucocele F. Nasal glial heterotopia

2. Which of the following is the most common subcuta-neous (extranasal) nasal mass in infancy? A. Encephalocele B. Dermoid C. Hemangioma D. Nasal glial heteretopia E. Rhabdomyosarcoma

3. Find the nonmatching pair. A. Nasal glioma; optic nerve glioma B. Dermal sinus; nasal pit C. Epidermoid cyst; bifid crista galli D. Dermoid; T1 hyperintensity E. Encephalocele; polymicrogyria

4. Surgical resection of a nasal glial heterotopia is the preferred treatment. True or false?

Page 68: Head and Neck Imaging - Case Review Series - Challenge

372

A N S W E R S

C A S E 1 8 3

Nasal Glial Heterotopia1. A, B, C, D, and F

2. C

3. A

4. True

ReferenceHedlund G. Congenital frontonasal masses: developmen-

tal anatomy, malformations and MR imaging. Pediatr Radiol. 2006;36:647–662.

Cross-ReferenceNeuroradiology: THE REQUISITES, 3rd ed, pp 422–

423.

CommentDevelopmental frontonasal mass lesions include nasal gliomas, encephaloceles, dermoid/epidermoid cysts, and dermal sinuses. Nasal glioma is a misnomer because these lesions are not neoplastic. They contain glial ele-ments and have no connection to the brain; hence a better term is nasal glial heterotopia. They typically present as subcutaneous masses along the dorsum of the nose or medial canthal region. About one third of nasal glial heteropias are intranasal, as seen in this case (see the figures), and are associated with the lateral nasal wall, the middle turbinate, or less commonly the nasal septum. On CT and MRI, nasal glial heteropias show solid tissue characteristics that resemble brain tissue, although T2 signal is usually higher in heterotopic tissue than it is in the brain. Cysts may be identified within the lesion. They typically do not show enhancement.

Frontonasal encephaloceles typically occur in the region of the glabella and clinically present as subdermal masses. Encephaloceles contain gliotic brain tissue and CSF. They show isointense T1 and hyperintense T2 signal compared to the brain. A connection between the brain and encephalocele is demonstrated, which helps differentiate encephaloceles from nasal glial heterotopias, which lack such a connection.

The nasal dermal sinus malformations develop along the course of the regressing embryologic dural tract, which extends from the tip of the nose to the foramen cecum just anterior to the crista galli. They may come in the form of dermoid or epidermoid cysts and sinus tracts. They are often subcutaneous but can be entirely intrana-sal. About 20% of them will have intracranial extension of the sinus tract or intracranial dermoid/epidermoid cysts. The most common form of this spectrum is a dermoid cyst occurring in the region of glabella.

Both the diagnosis and surgical planning hinge on whether the nasal mass is connected to the intracranial

compartment. Demonstration of the presence or absence of a small connection between a nasal mass and the brain requires high-resolution multiplanar images focused in the nasofrontal region. In general, MRI is more suitable than CT for this purpose because the incomplete ossification of the anterior skull base may make it difficult to differentiate a true defect from nonos-sified bone.

Other intranasal masses of infants include hemangio-mas, which follow the imaging features of hemangiomas seen elsewhere in the neck, and dacrocystoceles that develop secondary to distal obstruction of the nasolacri-mal duct and present as a cystic mass in the inferior meatus.

Notes

Page 69: Head and Neck Imaging - Case Review Series - Challenge

373

C A S E 1 8 4

History: A Japanese patient with bilateral nodular parotid enlargement

1. Which of the following conditions is in the differen-tial diagnosis of multiple solid masses in parotid glands? (Choose all that apply.) A. Warthin’s tumors B. Lymphadenopathy C. HIV lymphoepithelial lesions D. Oncocytomas

2. Which of the following conditions is the most common inflammatory condition in this space in adults? A. Sialadenitis B. Sialolithiasis C. Abscess D. Pleomorphic adenoma

3. Which of the following best describes Kikuchi’s disease? A. An inflammatory adenitis that is common in Japan B. A parotitis from a type of nematode C. Sialadenitis that mimics a neoplasm by its masslike

infiltration D. Sialolithiasis with ductal obstruction E. None of the above

4. Which of the following best describes Kimura’s disease? A. An inflammatory adenitis associated with sialade-

nitis that is common in Japan B. A parotitis from a type of nematode C. Sialadenitis that mimics a neoplasm by its masslike

infiltration D. Sialolithiasis with ductal obstruction E. None of the above

Page 70: Head and Neck Imaging - Case Review Series - Challenge

374

A N S W E R S

C A S E 1 8 4

Kimura’s Disease1. A, B, and C

2. B

3. A

4. A

ReferenceSun QF, Xu DZ, Pan SH, et al. Kimura disease: review

of the literature. Intern Med J. 2008 Aug;38(8):668–672.

Cross-ReferenceNeuroradiology: THE REQUISITES, 3rd ed, p 448.

CommentKimura’s disease is a rare inflammatory disease usually affecting Asian patients and more common in Japan and China than the United States. It is characterized by swell-ing of the salivary glands (see the figure) and neck nodes with a diffuse hypervascular adenopathy in the cervical chains favoring the submental and submandibular regions, blood eosinophilia, and a predilection for men aged 10 to 30 years. There is elevated IgE and the lymph nodes themselves may have eosinophilic infiltration as well. The pathogen for this has not been isolated. Para-sites or autoimmune reaction has been blamed.

The head and neck subcutaneous tissues are involved most commonly, and they evolve from poorly circum-scribed to hard lesions. The submandibular glands may be affected worse than parotid glands. On CT the nodes enhance and the salivary glands are enlarged. The lesion is hot on PET scanning.

Kikuchi’s disease is a histiocytic necrotizing lymphad-enitis presenting with cervical lymphadenopathy and fever. There are cutaneous lesions in 40%.

Notes

Page 71: Head and Neck Imaging - Case Review Series - Challenge

375

C A S E 1 8 5

History: A 52-year-old woman with rhinorrhea

1. Which of the following is the most accurate test in establishing the diagnosis of CSF leak in cases of rhinorrhea? A. Glucose, protein, and electrolyte content of

the fluid B. Presence of beta-2 transferrin in the fluid C. CT cisternogram D. Radionuclide cisternogram E. High-resolution MRI

2. Spontaneous CSF leak from the skull base is com-monly secondary to which of the following causes of increased intracranial pressure? A. Intracranial mass B. Meningitis C. Pseudotumor cerebri D. Hydrocephalus E. Chiari I malformation

3. Which of the following is the most common site of spontaneous CSF leak at the skull base? A. Temporal bone; tegmen tympani B. Lateral wall of the sphenoid sinus C. Posterior wall of the frontal sinus D. Cribriform lamella E. Pterygoid plate

4. High-resolution CT without intrathecal contrast has greater than 70% sensitivity for the detection of the site of CSF leak. True or false?

Page 72: Head and Neck Imaging - Case Review Series - Challenge

376

A N S W E R S

C A S E 1 8 5

Sphenoid Meningoencephalocele1. B

2. C

3. D

4. True

ReferenceTabaee A, Anand VK, Cappabianca P, et al. Endoscopic

management of spontaneous meningoencephalocele of the lateral sphenoid sinus. J Neurosurg. 2010 May;112(5):1070–1077.

Cross-ReferenceNeuroradiology: THE REQUISITES, 3rd ed, pp 431–

432.

CommentRhinorrhea is most commonly secondary to rhinitis. CSF leaks from the skull base may present with rhinorrhea as well. Beta-2 transferrin is a protein found only in the CNS. Detection of beta-2 transferrin in rhinorrhea fluid is highly specific for CSF leak. Most CSF leaks from the anterior or central skull base occur through the bony defects created by prior trauma or surgery, particularly endoscopic sinus surgery, transsphenoidal hypophysec-tomy, and craniofacial tumor resections. Patients with no history of trauma or surgery may occasionally develop spontaneous CSF leaks from the skull base. The identifi-cation of the site of leak from the skull base in cases of spontaneous CSF leaks may be challenging. High-resolution CT is usually obtained as the first imaging study to look for a bony defect. CT cisternogram, which involves administration of intrathecal iodinated contrast material, may be performed to positively demonstrate the site of leak. The precise anatomic information afforded by CT coupled with leakage of the contrast material provides a roadmap for surgical correction. Recently, high-resolution MRI has been used with success to identify sites of leak. Radionuclide cisternogram is usually reserved for cases when CSF leak is suspected, but it could not be confirmed because this modality provides a higher sensitivity but very limited anatomic detail. Because nuclear medicine scanning with indium DTPA can be performed for 24 to 48 hours, it may be useful for transient/intermittent leakers. The most common site of CSF leak from the skull base is the crib-riform lamella, which is the thinnest bone in the body and is also prone to injury during functional endoscopic sinus surgery (ethmoidectomy or turbinectomy) because one of the attachments of the middle turbinate is at the cribriform plate. The sphenoid sinus is also a frequent site of CSF leak in spontaneous CSF rhinorrhea. The

lateral inferior wall is the most frequent site in the sphe-noid sinus (see the figures). The reason for this is not clear; a potential defect created by the remnant of an embryologic structure named the lateral craniopharyn-geal canal (Sternberg’s canal) has been named as the culprit, but this has not been confirmed. Indentations of the arachnoid granulations (arachnoid pits) on the sphe-noid and temporal bones are frequent and may act as the initial site of the osteodural defects that give rise to encephalocele formation and CSF leaks. Regardless of the exact etiology, the lateral walls of the sphenoid sinus should be carefully scrutinized in cases of spontaneous CSF leaks. Also important is that some patients with spontaneous CSF leak have increased intracranial pres-sure, which if not addressed, may lead to recurrence of the CSF leak after surgical repair.

Notes

Page 73: Head and Neck Imaging - Case Review Series - Challenge

377

C A S E 1 8 6

History: Headaches

1. Which of the following tumor types is in the differ-ential diagnosis given the spread of the neoplasm? (Choose all that apply.) A. Adenoid cystic carcinoma B. Basal cell carcinoma C. Pleomorphic adenoma D. Rhabdomyosarcoma E. Lymphoma

2. What percentage of nasopharyngeal malignancies are adenoid cystic carcinomas? A. <3% B. 3–10% C. 11–20% D. >20%

3. Adenoid cystic carcinoma has a high rate of disease-free survival and overall survival. True or false?

4. All of the following statements about nasopharyngeal carcinoma are true EXCEPT: A. It is overrepresented as a head and neck carci-

noma in children. B. It is associated with Epstein-Barr virus. C. Type III is also known as lymphoepithelioma or

Schmincke’s tumor and is dominated by undiffer-entiated cells.

D. It has a bimodal age distribution, except in endemic regions, such as Southeast Asia.

E. Type II disease, keratinizing squamous cell carci-noma, is associated with alcohol use.

Page 74: Head and Neck Imaging - Case Review Series - Challenge

378

A N S W E R S

C A S E 1 8 6

Nasopharyngeal Adenoid Cystic Carcinoma1. A and E

2. A

3. False

4. E

ReferencesLiu TR, Yang AK, Guo X, et al. Adenoid cystic carci-

noma of the nasopharynx: 27-year experience. Laryngoscope. 2008 Nov;118(11):1981–1988.

Marcus KJ, Tishler RB. Head and neck carcinomas across the age spectrum: epidemiology, therapy, and late effects. Semin Radiat Oncol. 2010 Jan;20(1):52–57.

Cross-ReferenceNeuroradiology: THE REQUISITES, 3rd ed, p 458.

CommentNasopharyngeal adenoid cystic carcinoma is an unusual histology to be expressed in this location, representing less than 3% of the overall nasopharyngeal malignancies. It retains its predilection for perineural spread in this location, seen in 27% of cases (as opposed to 55% for all sites for adenoid cystic carcinoma) (see the figures). When spreading from this location it may find the orbit or the intracranial compartment and is not averse to affecting the trigeminal and oculomotor nerves. Local invasion to the skull base, parapharyngeal space, ptery-gopalatine fossa, and sinonasal cavity is common as well. As opposed to nasopharyngeal carcinoma, adenoid cystic carcinoma is relatively radiation-insensitive. Therefore, surgical treatment may be contemplated. Ten-year sur-vival rates may be as high as 60% to 90%, but usually it is not disease-free. It’s just that adenoid cystic carcinoma is not that aggressive and metastasizes late.

Nasopharyngeal carcinoma is classified by the WHO as type I squamous cell carcinoma, moderate well-differentiated keratin-producing; type II nonkeratinizing carcinoma with mature to anaplastic cells; and type III undifferentiated cells, including lymphoepithelioma, anaplastic, clear cell, and spindle cell variants. Type I is associated with smoking and usually presents in adult-hood. Types II and III may be seen in adults and are associated with EBV exposure as well as other genetic predispositions. In children type III disease dominates; the exposure to EBV is paramount. For kids the disease is usually far advanced at presentation (likely due to delay in diagnosis). Because lymphadenopathy and ear infections are often felt to be infectious in etiology, EBV titers may be used to monitor disease. The most common presentation in adults is also a nodal mass in the neck or ear congestion/hearing loss. Cranial neuropathies occur

in 20% of adults with nasopharyngeal carcinoma. Treat-ment is dominated by high-dose radiotherapy and con-current or adjuvant chemotherapy. Children tend to do better than adults.

The rate of nasopharyngeal cancers in patients from Singapore is 17× higher than that in the United States. Children are more commonly affected in the United States, Israel, and Uganda (>10% of all patients) but are less than 1% to 2% of nasopharyngeal carcinomas in China and Turkey. MRI is the best means to evaluate these patients due to the perineural spread, which may occur, and also to detect intracranial extension.

Notes

Page 75: Head and Neck Imaging - Case Review Series - Challenge

379

C A S E 1 8 7

History: A 69-year-old with acute-onset severe respira-tory distress that required emergent tracheostomy

1. Which of the following best characterizes the findings on this study? A. Mass in uvula B. Tongue mass C. Epiglottic mass D. False vocal cord mass E. None of the above

2. Which of the following disorders is the most likely diagnosis? A. Epiglottitis B. Diffuse mucosal lymphoma C. Diffuse mucosal hemorrhage D. Angioedema E. Amyloidosis

3. Causes of angioedema include all of the following EXCEPT: A. ACE inhibitors B. NSAIDs C. Sarcoidosis D. Lymphoma E. SLE

4. All of the following statements regarding angioedema are true EXCEPT: A. It develops as a result of an IgE-mediated allergic

reaction. B. It develops secondary to C1 inhibitor deficiency. C. It is limited to the upper aerodigestive tract. D. It is hereditary. E. It is recurrent.

Page 76: Head and Neck Imaging - Case Review Series - Challenge

380

A N S W E R S

C A S E 1 8 7

Angioedema Secondary to ACE Inhibitor1. E

2. D

3. C

4. C

ReferenceByrd JB, Adam A, Brown NJ. Angiotensin-converting

enzyme inhibitor-associated angioedema. Immunol Allergy Clin North Am. 2006 Nov;26(4):725–737.

Cross-ReferenceNeuroradiology: THE REQUISITES, 3rd ed, p 451.

CommentAngioedema is a disease associated with sudden attacks of swelling of the deep cutaneous and mucosal/submucosal tissues due to a transient increase in endo-thelial permeability in the capillaries. It can be fatal if the airway is compromised. Involvement of the upper aerodigestive tract may be isolated or a part of systemic involvement. Allergic (IgE-mediated) and nonallergic (non–IgE-mediated) forms of angioedema are recog-nized. Nonallergic angioedema can be hereditary (C1 esterase inhibitor deficiency), acquired (secondary to lymphoma, other malignancies, and systemic disorders), related to ACE inhibitor therapy, or idiopathic.

ACE inhibitor-related angioedema is a relatively fre-quent cause of angioedema that can present months or years after the initiation of therapy, although most cases occur within weeks after the first dose of ACE inhibitor. CT findings include diffuse swelling of the mucosa and submucosal tissue in the upper aerodigestive tract with or without narrowing of the airway, depending on the severity of edema (see the figures). Differential diagnosis includes epiglottitis and laryngitis, but the involvement is more localized in these situations. Skin, subcutaneous tissue, tongue, and other soft-tissue structures in the neck can also demonstrate diffuse or regional swelling. Make sure you exclude an anaphylactic reaction to the iodinated contrast you administered for the case!

Notes

Page 77: Head and Neck Imaging - Case Review Series - Challenge

381

C A S E 1 8 8

History: Multiple neck masses

1. Bilateral non-necrotic massive lymphadenopathy is not uncommon in which of the following disorders? (Choose all that apply.) A. HIV B. Lymphoma C. Squamous cell carcinoma D. Mycobacterial infection E. Sinus histiocytosis

2. Which of the following characteristics is true of Rosai-Dorfman disease? A. Favors adults B. Has massive lymphadenopathy C. Rarely has extranodal disease D. Is associated with hypercalcemia E. Is related to EBV exposure

3. Post-transplant lymphoproliferative disorder (PTLD) has the highest incidence after which of the following transplants? A. Bone marrow transplantation B. Brain transplantation C. Liver transplantation D. Lung transplantation E. Renal transplantation

4. Which of the following represents the most common head and neck site of extranodal Rosai-Dorfman disease? A. Brain B. Sinonasal C. Salivary glands D. Masticator space E. Orbit

Page 78: Head and Neck Imaging - Case Review Series - Challenge

382

A N S W E R S

C A S E 1 8 8

Rosai-Dorfman Disease: Sinus Histiocytosis with Massive Lymphadenopathy1. B and E

2. B

3. D

4. B

ReferenceLa Barge DV 3rd, Salzman KL, Harnsberger HR, et al.

Sinus histiocytosis with massive lymphadenopathy (Rosai-Dorfman disease): imaging manifestations in the head and neck. AJR Am J Roentgenol. 2008 Dec;191(6):W299–W306.

Cross-ReferenceNeuroradiology: THE REQUISITES, 3rd ed, pp 472, 498,

500.

CommentRosai-Dorfman disease is also called sinus histiocytosis with massive lymphadenopathy. It is a disease of youngsters (80% < 20 years old and 67% < 10 years old) and causes bilateral massive cervical lymphadenopathy. As such it is in the differential diagnosis with mononu-cleosis, cat scratch fever, lymphoma, and mastocytosis, which also affect younger patients. The patients may have fevers, elevated erythrocyte sedimentation rates, and polyclonal hypergammaglobulinemia.

Imaging features are that of large non-necrotic lymph-adenopathy (see the figure). The nodes are usually hot on gallium and PET scanning. Extranodal deposits occur in approximately half of patients and may be in the skin, sinonasal cavity, salivary glands, orbits, and bones. Hypointensity of sinonasal sinus histiocytosis on T2W imaging calls up a differential diagnosis of lymphoma, sarcoidosis, fungal disease, pseudotumor, and granulo-matous infections. The most common manifestation in the orbit is an extraconal mass causing proptosis.

Intracranially Rosai-Dorfman disease may infiltrate the dura of the sella, cavernous sinus, periclival regions, and foramen magnum. The dura around venous sinuses may also be affected.

Treatment is watchful waiting at first. Often the disease is self-limited. However interferon, steroids, che-motherapy, or radiation may be utilized for advanced disease. Surgery is confined to those patients with pro-gressive, obstructive disease.

Notes

Page 79: Head and Neck Imaging - Case Review Series - Challenge

383

C A S E 1 8 9

History: A 17-year-old woman with headaches and visual disturbance

1. All of the following disorders should be included in the differential diagnosis EXCEPT: A. Osteoma B. Osteblastoma C. Fibrous dysplasia D. Osteosarcoma

2. Damage to which of the following nerves is associ-ated with sphenoid sinus masses or sphenoid sinus surgery? A. Olfactory nerve B. Optic nerve C. Oculomotor nerve D. Trochlear nerve E. Abducens nerve

3. Why is the vidian canal an important landmark for the endoscopist? A. It marks the lateral wall of the sphenoid sinus. B. The carotid artery is always lateral to the vidian

canal. C. It is symmetric on both sides. D. It is 5 mm medial to the cavernous sinus.

4. The foramen rotundum is always lateral to the vidian canal. True or false?

Page 80: Head and Neck Imaging - Case Review Series - Challenge

384

A N S W E R S

C A S E 1 8 9

Osteoblastoma1. C

2. B

3. B

4. True

ReferenceLee EJ, Park CS, Song SY, Park NH, Kim MS. Osteoblas-

toma arising from the ethmoidal sinus. AJR Am J Roentgenol. 2004 May;182(5):1343–1344.

Cross-ReferenceNeuroradiology: THE REQUISITES, 3rd ed, p 433.

CommentOsteoblastomas are rare benign bone tumors that can be locally aggressive. Sinonasal osteoblastomas are rare and usually occur in the ethmoid and sphenoid sinuses. Bone-forming tumors are easily recognized on CT on the basis of extremely hyperdense osteoid matrix, which is similar in attenuation to cortical bone (see the figures). This is different from fibrous dysplasia, which approxi-mates the attenuation of medullary bone because of sig-nificant fibrotic component. The most common sinonasal bone-forming tumor is osteoma, which most commonly occurs in the ethmoid and frontal sinuses as an incidental finding. They are often smaller than 1 cm and, if critically located, can cause obstruction of sinus drainage path-ways leading to sinusitis or mucoceles. Most osteomas are uniformly very dense, but occasionally they may contain less dense areas due to fibrous matrix. Osteoblas-tomas are similar in density to osteomas, but they are larger and cause expansion of the sinus in which they are located, although they typically lack more aggressive features such as bone destruction and invasion of adja-cent spaces. Some osteoblastomas can be predominantly fibrotic rather than osseous, in which case differentia-tion from other bone-forming sinonasal tumors such as fibrous dysplasia and ossifying fibroma may be difficult. At the malignant end of the spectrum of bone-forming tumors is osteosarcoma, which rarely occurs in the sino-nasal cavity. Spiculated new bone formation with destruction of the surrounding structures allows differ-entiation of osteosarcoma from other bone-forming masses. Chondrosarcoma can also form dense calcifica-tions, although these usually look different than typical osteoid formation.

Among the potential complications of sphenoid sinus surgery, optic nerve and carotid artery injury are the two most important. Optic nerves and carotid arteries can bulge into the sphenoid sinus and lack a strong bony coverage, which makes them vulnerable. The

vidian canal serves as an important landmark due to its constant relationship to the carotid artery. By not drilling lateral to the vidian canal, the endoscopist can avoid carotid artery penetration.

Notes

Page 81: Head and Neck Imaging - Case Review Series - Challenge

385

C A S E 1 9 0

History: Congestion

1. What are the most common three sarcomas to affect the sinonasal cavity? (Choose all that apply.) A. Osteosarcoma B. Rhabdomyosarcoma C. Chondrosarcoma D. Synovial sarcoma E. Fibrosarcoma

2. Which of the following is a frequent finding in syno-vial sarcomas? A. Joint origin B. Hemorrhage C. Whorls of calcification D. Fatty degeneration E. None of the above

3. Why do extra-articular synovial sarcomas arise? A. Remnant synovial cells are present outside the

joint. B. They arise from embryologic rests. C. Synovial cells are pleuripotential. D. The cell of origin is not from the synovium. E. They all arise from joints.

4. Which of the following locations in the aerodigestive system is the most common site of synovial sarcoma? A. Sinonasal B. Larynx C. Nasopharynx D. Oropharynx E. Hypopharynx

Page 82: Head and Neck Imaging - Case Review Series - Challenge

386

A N S W E R S

C A S E 1 9 0

Sinus Synovial Sarcoma1. A, B, and C

2. B

3. D

4. E

ReferenceO’Sullivan PJ, Harris AC, Munk PL. Radiological features

of synovial cell sarcoma. Br J Radiol. 2008 Apr;81(964):346–356. Epub 4 Feb 2008.

Cross-ReferenceNeuroradiology: THE REQUISITES, 3rd ed, p 438.

CommentSynovial sarcomas are extremely enigmatic lesions. Most occur in the extremities, with only 3% in the head and neck. However, they can occur anywhere in the head and neck, most commonly in the parapharyngeal and masticator space, and they have a highly heterogeneous appearance. One can see cystic, hemorrhagic (40%), and calcific (27%) change as well as necrosis. Punctate and coarse calcifications have been described. 20% of lesions have fluid–fluid levels. Multiloculation and internal septa-tion is the rule.

The average age at diagnosis is approximately 30 years. The lesions are usually painful. The 5-year survival is 60%, which is not bad for a sarcoma. 25% develop metastases, usually to the lungs, and there is a high rate of recurrence.

This is a highly unusual site of occurrence of synovial sarcoma (see the figure). They affect the soft tissue more than the mucosal space and when affecting a mucosal surface they arise more commonly in the hypopharynx. Variants of synovial sarcoma are subclassified into 4 types: (1) biphasic type with epithelial and spindle cell components, (2) monophasic spindle cell type with little or no evidence of epithelial differentiation, (3) monopha-sic epithelial type, and (4) poorly differentiated.

Notes

Page 83: Head and Neck Imaging - Case Review Series - Challenge

387

C A S E 1 9 1

History: A 48-year-old woman with bilateral arm weak-ness and numbness

1. Which of the following disorders is the most likely diagnosis? A. Viral neuropathy B. Entrapment syndrome C. Polyneuropathy D. Lymphoma E. Stretch injury

2. All of the following statements about chronic inflam-matory demyelinating polyneuropathy (CIDP) are true EXCEPT: A. CIDP is the most common form of acquired demy-

elinating polyneuropathies. B. CIDP involves the lumbosacral plexus more com-

monly than the brachial plexus. C. CIDP diagnosis is based on its characteristic MRI

findings. D. CIDP can involve the cauda equina. E. CIDP can involve the cranial nerves.

3. Which of the following conditions can be associated with CIDP? A. HIV B. Diabetes C. Lymphoma D. Inflammatory bowel disease E. All of the above

4. Other names given to CIDP include multifocal acquired demyelinating sensory and motor neuropa-thy (MADSAM), idiopathic hypertrophic brachial neu-ropathy, focal brachial conduction block in idiopathic brachial plexopathy, and multifocal motor neuropa-thy with conduction block (MMNCB). True or false?

Page 84: Head and Neck Imaging - Case Review Series - Challenge

388

A N S W E R S

C A S E 1 9 1

Chronic Inflammatory Demyelinating Polyneuropathy1. C

2. C

3. E

4. False

ReferenceSaperstein DS. Chronic acquired demyelinating polyneu-

ropathies. Semin Neurol. 2008 Apr;28(2):168–184.

Cross-ReferenceNeuroradiology: THE REQUISITES, 3rd ed, p 503.

CommentCIDP is the most common form of a group of poorly understood acquired polyneuropathy syndromes. Ac-quired polyneuropathies are very common, and their etiology is diverse. Demyelinating polyneuropathies occur as a result of an immune-mediated insult to the peripheral nerves and consist of many clinically distinct entities, although their classification is evolving with better understanding of various immunologic mecha-nisms and clinical features. CIDP appears to be the pro-totype of acquired demyelinating polyneuropathies. CIDP can be seen at any age, although it is most common in the 5th and 6th decades. It usually has a progressive course that includes episodes of demyelination and remyelination, which are characterized by marked inflammatory reaction and nerve swelling. Bilateral and relatively symmetric involvement of multiple nerves and plexuses is typical (see the figure), although isolated or more pronounced involvement of the brachial plexus may occur. MRI shows bilateral diffuse thickening of the nerves of the brachial plexus, which shows elevated T2 signal. Postcontrast enhancement may or may not be present, depending on the phase of the inflammatory process. A diagnosis of a polyneuropathy should be offered with these findings, although markedly enlarged nerves clumped together may mimic a mass lesion, par-ticularly when involvement is asymmetric.

Notes

Page 85: Head and Neck Imaging - Case Review Series - Challenge

389

C A S E 1 9 2

History: Numbness and tingling and weakness in the left arm

1. Which of the following describes what is happening in this case? A. Hematoma has developed in the muscle. B. Contrast has been injected into the brachial

plexus. C. Glue has been injected into an AVM. D. There has been a failed attempt at facet block. E. Contrast has been injected into the anterior

scalene muscle.

2. Which of the following is the purpose of this procedure? A. It ablates the lesion. B. It blocks the brachial plexus. C. It paralyzes the muscle. D. It determines if relaxation of the scalene muscles

will enlarge the thoracic outlet and relieve symptoms.

E. All of the above

3. Which of the following surgeries are performed for thoracic outlet syndrome? A. Scalenectomy B. Manubrium resection C. Vascular bypass D. Nerve grafting E. Hypophysectomy

4. Where is the brachial plexus in relation to the ante-rior scalene muscle? A. Anterior B. Posterior C. Inferior D. Superior E. None of the above

Page 86: Head and Neck Imaging - Case Review Series - Challenge

390

A N S W E R S

C A S E 1 9 2

Scalene Block1. E

2. D

3. A

4. B

ReferenceBraun RM, Sahadevan DC, Feinstein J. Confirmatory

needle placement technique for scalene muscle block in the diagnosis of thoracic outlet syndrome. Tech Hand Up Extrem Surg. 2006 Sep;10(3):173–176.

Cross-ReferenceNeuroradiology: THE REQUISITES, 3rd ed, p 503.

CommentThoracic outlet syndrome may cause numbness and tingling in the upper extremity. Weakness may also be present. Symptoms are worse when raising the arm above the head, after exercise, at night, or when using the arm for activities such as combing one’s hair or driving a car.

Thoracic outlet syndrome has many etiologies. These include cervical ribs, aberrantly large first or second ribs, apophysomegaly, transverse process enlargement, and fibrous bands. The anterior scalene muscle, by virtue of fibrosis, trauma, hematoma, or hypertrophy, may also be a source of compression of either the subclavian artery or the brachial plexus. Pectoralis minor syndrome leads to pain in the anterior chest wall associated with tho-racic outlet syndrome. Usually a traumatic event precipi-tates the onset of pectoralis minor or thoracic outlet syndrome.

The scalene block is a therapeutic prognostic indica-tor test used to determine whether removal or paralysis of the anterior scalene muscle will benefit the patient. The scalene muscle arises from the cervical spine and inserts on the first rib. If, on administration of anesthetic, the symptoms resolve, it suggests a potential benefit from scalenectomy surgery or intramuscular Botox injec-tion. Spread of the anesthetic into the brachial plexus, out of the muscle, can lead to false-positive scalene block results. This is why contrast material is injected first to demonstrate lack of extramuscular spread before anes-thetic injection (see the figure).

Notes

Page 87: Head and Neck Imaging - Case Review Series - Challenge

391

C A S E 1 9 3

History: A 30-year-old with headache

1. Which of the following disorders should be included in the differential diagnosis? (Choose all that apply.) A. Mucocele B. Cholesterol granuloma C. Epidermoid D. Langerhans cell histiocytosis E. Fibrous dysplasia

2. Which of the following lesions would have enhance-ment on MRI? A. Mucocele B. Epidermoid C. Cholesterol granuloma D. Fibrous dysplasia

3. Epidermoid tumors do not show increased signal on DWI in which of the following locations? A. Intracranial B. Calvarial C. Temporalis fossa D. Middle ear E. None

4. Lateral frontal sinus and supraorbital ethmoid muco-celes often require a surgical approach different from the endoscopic intranasal approach. True or false?

Page 88: Head and Neck Imaging - Case Review Series - Challenge

392

A N S W E R S

C A S E 1 9 3

Lateral Frontal Sinus Mucocele1. A, B, C, and D

2. D

3. E

4. True

ReferenceChiu AG, Vaughan WC. Management of the lateral frontal

sinus lesion and the supraorbital cell mucocele. Am J Rhinol. 2004 Mar-Apr;18(2):83–86.

Cross-ReferenceNeuroradiology: THE REQUISITES, 3rd ed, pp 339, 431.

CommentThe orbital plate of the frontal bone may or may not be pneumatized. Air cells in the orbital plate of the frontal bone are either of frontal sinus or ethmoid sinus (supra-orbital ethmoid cell) origin. Diseases of the lateral frontal sinus and the supraorbital ethmoid cells may be difficult to address with endoscopic approaches. Occasionally, a sequestered air cell (an air cell that has no apparent con-nection with the rest of the frontal or ethmoid cells on high-resolution CT) in the lateral frontal bone may give rise to a mucocele (see the figures), which often creates a diagnostic problem. The imaging features of these resemble mucoceles seen elsewhere in the sinonasal cavity (i.e., expansion of the air cell with thinning of the bony walls). Most lesions seen in the orbital plate repre-sent one of three benign entities: mucocele, cholesterol granuloma, and epidermoid, which may look alike on CT. MRI allows differentiation; markedly increased T1 signal in cholesteatomas, increased DWI signal in epider-moids, and variable T1 and T2 signal in mucoceles. None of these show enhancement on MRI. When there is enhancing soft tissue in an expansile bone lesion in the orbital plate, more aggressive lesions such as Langerhan cell hystiocytosis, plasmacytoma, and metastasis should be considered. Frank bone destruction is also suspicious for aggressive processes. Fibrous dysplasias have variable signal characteristics and often show prominent enhance-ment. Matrix formation (ground-glass appearance) within the lesion is seen on CT.

Notes

Page 89: Head and Neck Imaging - Case Review Series - Challenge

393

C A S E 1 9 4

History: Headaches, congestion in an elderly man

1. Which of the following disorders is in the differential diagnosis of this sinus mass? (Choose all that apply.) A. Squamous cell carcinoma B. Sinonasal undifferentiated carcinoma (SNUC) C. Melanoma D. Olfactory neuroblastoma E. Rhabdomyosarcoma

2. Which of the following imaging findings appear in the study? A. Intracranial spread B. Intraorbital spread C. Obstructed secretions D. Bone destruction E. All of the above

3. Which of the following is the unusual feature of SNUC tumors as opposed to other sinonasal cancers? A. Early metastases B. Retropharyngeal lymphadenopathy C. Dural/leptomeningeal spread D. Perineural spread E. Subarachnoid seeding

4. Which of the following percentages represents the 5-year survival for SNUC? A. 0–20% B. 21–40% C. 41–60% D. 61–80% E. >80%

Page 90: Head and Neck Imaging - Case Review Series - Challenge

394

A N S W E R S

C A S E 1 9 4

Sinonasal Undifferentiated Carcinoma1. A, B, and D

2. E

3. C

4. A

ReferenceEnepekides DJ. Sinonasal undifferentiated carcinoma:

an update. Curr Opin Otolaryngol Head Neck Surg. 2005 Aug;13(4):222–225.

Cross-ReferenceNeuroradiology: THE REQUISITES, 3rd ed, p 436.

CommentSNUC are highly aggressive lesions of the sinonasal cavity that favor males over females by a 2.5:1 ratio. They occur in older individuals with complaints of nasal con-gestion, epistaxis, diplopia, pain, and headache. The symptoms are rapidly progressive. They are relatively rare cancers of the sinonasal cavity; 60% to 70% of all sinonasal cancers are squamous cell carcinomas, 10% to 20% adenocarcinomas, and the rest are split among esthesioneuroblastomas, adenoid cystic carcinoma, other salivary gland origin tumors, melanoma, lym-phoma, and rhabdomyosarcomas. Wood dust, smoking, formaldehyde exposure, nickel, chromium, and asbestos are risk factors for sinonasal cancers.

On imaging, SNUCs are large tumors that often span one or more sinonasal regions. Intraorbital and intracra-nial spread at presentation is common (see the figures). A recent review reported dural invasion in 50%, cavern-ous sinus spread in 30%, periorbital invasion in 43%, and orbital invasion in 30%. Nodal disease, however, is present in only 10% to 30% of patients.

Treatment is usually the full-scale bombardment of surgery, chemotherapy, and radiation therapy. It is rela-tively radioresistant, however. Two-year survival rates are usually quoted as 40% to 50% with very few 5-year survivors disease-free.

Notes

Page 91: Head and Neck Imaging - Case Review Series - Challenge

395

C A S E 1 9 5

History: A 48-year-old man with left-sided conductive hearing loss

1. Which of the following is the most common primary tumor of the middle ear? A. Middle ear adenoma B. Paraganglioma C. Hamartoma D. Meningioma E. Schwannoma

2. Which of the following locations is characteristic of glomus tumors in the middle ear? A. Sinus tympani B. Medial wall C. Tegmen tympani D. Oval window E. Round window

3. All of the following statements regarding middle ear adenomas are true EXCEPT: A. Most common presentation is conductive

hearing loss B. Well circumscribed C. Vascular D. Benign E. Low recurrence rate once completely removed

4. Octreotide scintigraphy’s sensitivity for paraganglio-mas is greater than 90%. True or false?

Page 92: Head and Neck Imaging - Case Review Series - Challenge

396

A N S W E R S

C A S E 1 9 5

Middle Ear Adenoma1. B

2. B

3. D

4. True

ReferenceBerns S, Pearl G. Middle ear adenoma. Arch Pathol Lab

Med. 2006 Jul;130(7):1067–1069.

Cross-ReferenceNeuroradiology: THE REQUISITES, 3rd ed, p 397.

CommentMiddle ear adenomas are rare tumors, which are usually well-circumscribed and easily resectable. They can have neuroendocrine and epithelial differentiation on histo-pathologic evaluation, which may lead to varying diag-noses. It is now understood that the previously used terms, including ceruminoma, ceruminous adenoma, monomorphic adenoma, and carcinoid tumor, all refer to the same tumor: middle ear adenoma with mixed pat-terns of differentiation.

Middle ear adenomas appear as well-defined soft-tissue masses on CT with no significant bone erosion, although they may surround and destroy the middle ear ossicles (see the figures). On MRI, they usually show decreased T1 signal, increased T2 signal, and enhance-ment with contrast.

The differential diagnosis of soft-tissue masses con-fined to the middle ear includes chronic otitis media, cholesteatoma (acquired and congenital), paragangli-oma (glomus tympanicum), middle ear adenoma, schwannoma, meningioma, choristoma/hamartoma, and schneiderian-type mucosal papilloma. A variety of skull base and temporal bone tumors can secondarily involve the middle ear, although these differ greatly in their size and extent compared to small middle ear masses. Octreotide scintigraphy is very sensitive for paraganglio-mas and may be useful in detecting unsuspected addi-tional foci, although high-resolution MRI and CT are essential in determination of the extent of tumor. Middle ear adenomas can be positive on octreotide scintigra-phy due to their neuroendocrine differentiation.

Notes

Page 93: Head and Neck Imaging - Case Review Series - Challenge

397

C A S E 1 9 6

History: Dwarfism

1. Chondrodysplasia punctata has which of the follow-ing characteristics? (Choose all that apply.) A. High incidence of cleft palate B. Multiple stippled epiphyses C. Foci of hyperdensity in respiratory cartilage D. Dwarfism

2. Which of the following is the best diagnosis in this case? A. Chondrodysplasia punctata B. Chondroectodermal dysplasia C. Enchondromatosis D. Campomelic dwarfism

3. Chondroectodermal dysplasia occurs in Amish com-munities and is associated with wormian bones in the skull. True or false?

4. Which of the following percentages represents the rate of sarcomatous transformation in Ollier’s disease? A. 0–20% B. 21–40% C. 41–60% D. 61–80% E. >80%

Page 94: Head and Neck Imaging - Case Review Series - Challenge

398

A N S W E R S

C A S E 1 9 6

Chondrodysplasia Punctata1. A, B, C, and D

2. A

3. True

4. B

ReferenceMundinger GS, Weiss C, Fishman EK. Severe tracheo-

bronchial stenosis and cervical vertebral subluxation in X-linked recessive chondrodysplasia punctata. Pediatr Radiol. 2009 Jun;39(6):625–628.

Cross-ReferenceNeuroradiology: THE REQUISITES, 3rd ed, p 446.

CommentChondrodysplasia punctata is also known as congenital stippled epiphyseal syndrome; it is a disorder associated with rhizomelic dwarfism. It is most commonly an auto-somal recessive disorder associated with congenital heart defects, cleft palate, distal phalangeal hypoplasia, midface hypoplasia, mental retardation, and death before age 2 years. The disease is associated with multiple punc-tate hyperdensities found in the epiphyses of the extrem-ities and axial skeleton; however, one can also see these same fine calcifications in the trachea and bronchi (see the figures). Clefts in the vertebral bodies are also characteristic.

A genetic defect in PEX7, which encodes the recep-tor for a subset of peroxisomal matrix enzymes affecting vitamin K–dependent arylsulfatase, is associated with the entity and has been mapped to chromosome 6. There are depressed levels of red cell plasmalogens, but pro-gressive accumulation of phytanic acid.

The most common manifestation of the disorder is cataracts, seen in more than 70% of cases; ichthyosis is also prevalent. Tracheal stenosis, as in this case, is another reported complication of the disease. Flat facies, well-demonstrated on this sagittal reconstruction, is another feature of the disorder.

Notes

Page 95: Head and Neck Imaging - Case Review Series - Challenge

399

C A S E 1 9 7

History: A 4-month-old with arm weakness since birth

1. Which of the following should be included in the dif-ferential diagnosis? (Choose all that apply.) A. Muscular dystrophy B. Corticospinal degeneration C. Nerve avulsion D. Moebius syndrome E. Sprengel’s deformity

2. Injury to which part of the brachial plexus at birth requires a different surgical approach? A. Preganglionic segment B. Postganglionic spinal segment C. Brachial plexus trunks D. Brachial plexus divisions E. Brachial plexus cords

3. All of the following statements about brachial plexus birth injury are true EXCEPT: A. Pseudomeningocele implies nerve root avulsion. B. Absence of pseudomeningocele rules out nerve

root avulsion. C. Injured postganglionic nerves show increased T2

signal. D. C5 and C6 are the most commonly injured nerve

roots. E. Post-traumatic neuromas are rarely identified

on MRI.

4. All of the following statements about brachial plexus birth injury are true EXCEPT: A. The prognosis for C5–C6 (Erb’s palsy) injuries is

better than for C5–T1 (flail arm) injuries. B. Klumpke’s palsy is the least common type of

injury. C. Even avulsed roots have a significant potential for

spontaneous recovery. D. CT/myelography is the gold standard for evalua-

tion of cervical root avulsion.

Page 96: Head and Neck Imaging - Case Review Series - Challenge

400

A N S W E R S

C A S E 1 9 7

Brachial Plexus Birth Palsy1. C

2. A

3. B

4. C

ReferenceHale HB, Bae DS, Waters PM. Current concepts in the

management of brachial plexus birth palsy. J Hand Surg Am. 2010 Feb;35(2):322–331.

Cross-ReferenceNeuroradiology: THE REQUISITES, 3rd ed, p 503.

CommentInjury to the brachial plexus during birth is a relatively common problem. Luckily, the majority of the injuries are mild and recover completely without intervention. Some patients have incomplete recovery, and some do not recover at all. The timing of surgical intervention is critical because delayed surgical repair after muscle atrophy occurs is associated with poorer outcomes, whereas early surgery increases the number of unneces-sary surgeries for patients who would spontaneously recover. It is generally believed that if no recovery occurs by age 3 months it is unlikely that any recovery will occur unless surgical repair is performed. To determine the type of surgery needed, one needs to know whether the injury affected the preganglionic or postganglionic plexus. Preganglionic injuries present as complete or partial avulsion of the cervical roots from the cord proxi-mal to the dorsal root ganglion, are associated with a worse prognosis, and require more sophisticated nerve transfer surgeries, whereas if the cervical roots are intact, local repair or grafting of the injured postganglionic segment in the supraclavicular fossa could be performed. Although it is important to identify the level and degree of postganglionic brachial plexus injuries, the primary goal of imaging is to evaluate for cervical root avulsion from the cord. Pseudomeningocele, a dilatation of the thecal sac ipsilateral to the avulsion, is an indirect sign of root avulsion and can be identified on routine cervical spine MRI. Pseudomeningocele is seen in 80% of all avul-sions, although lack of pseudomeningocele does not rule out root avulsion. Because of this one has to look at the continuity of the roots with cervical cord with high-resolution imaging; CT/myelography has been the gold standard for this purpose. Recently-high resolution T2W images have shown promise in providing similar infor-mation. The avulsed nerve roots usually coil over them-selves and form a nubbin along the anterior aspect of the cord (see the figures). CT is useless in evaluation of

postganglionic injuries. MRI shows thickening of the injured nerves in the brachial plexus. Depending on the avulsed nerve roots the clinical presentation varies; the most commonly injured nerves are the C5 and C6 roots, associated with proximal arm weakness (Erb’s palsy). The second most common type of injury involves all roots of the cervical spine (flail arm) and presents with weakness of the proximal as well as distal muscle groups, whereas the rare isolated C7 and C8 injuries (Klumpke’s palsy) manifest as intrinsic hand muscle weakness.

Notes

Page 97: Head and Neck Imaging - Case Review Series - Challenge

401

C A S E 1 9 8

History: Pain in the right maxilla

1. Which of the following is the typical location of a globulomaxillary cyst? A. Between medial incisors B. Between medial and lateral incisors C. Between lateral incisor and canine D. Between canine and premolar E. All of the above

2. Which of the following is the typical location of a nasolabial cyst? A. Between medial incisors B. Between medial and lateral incisors C. Between lateral incisor and canine D. Between canine and premolar E. None of the above

3. Which of the following is the typical location of a lateral periodontal cyst? A. Between molar teeth B. Between premolar teeth C. Between lateral incisor and canine D. Between canine and premolar E. None of the above

4. Which of the following is the typical location of a Stafne bone cyst? A. Along molar teeth B. Between premolar teeth C. Between lateral incisor and canine D. Between canine and premolar E. None of the above

Page 98: Head and Neck Imaging - Case Review Series - Challenge

402

A N S W E R S

C A S E 1 9 8

Globulomaxillary Cyst1. C

2. E

3. B

4. A

ReferenceYoshiura K, Weber AL, Runnels S, Scrivani SJ. Cystic

lesions of the mandible and maxilla. Neuroimaging Clin North Am. 2003 Aug;13(3):485–494.

Cross-ReferenceNeuroradiology: THE REQUISITES, 3rd ed, pp 450–

451.

CommentThe globulomaxillary cyst was originally described as a fissural cyst between the medial nasal and maxillary plates, interrupting fusion. It is exclusively seen affecting the maxillary teeth, classically between the roots of the lateral incisor and the canine tooth (see the figures). More evidence suggests this is of odontogenic origin because the cyst may be associated with unerupted teeth or may be in a periapical location. These are more often being called keratocysts now. Classic descriptions are of an inverted pear shape, as displayed on the coronal scan. They may become infected and may lead to divergence of neighboring teeth.

The nasolabial cyst is a cyst arising at the junction of the medial nasal processes, lateral nasal process, and maxillary process. The latest theory is that they arise from the termination of the nasolacrimal duct. They usually grow submucosally and are more common in women and in adulthood. They are pure cysts on MRI with no enhancement and are of CSF intensity. They are most often seen at the base of the nostril extending to the upper lip. Ten percent are bilateral. Treatment is excision.

The lateral periodontal cyst occurs between cuspid and bicuspid premolar teeth and is another fissural cyst. It is unilocular and will splay the roots of premolar teeth. Lateral periodontal cysts occur in adults and are more common in the mandible.

A Stafne bone cyst is an ovoid indentation made by the submandibular gland along the inner cortex of the man-dible below the inferior alveolar canal and therefore below the roots of the teeth. It is usually identified at the mandibular molar region posteriorly. This was initially thought to be a pathologic process, but is now recog-nized as a normal variant. It is thinned bone effectively.

Notes

Page 99: Head and Neck Imaging - Case Review Series - Challenge

403

C A S E 1 9 9

History: A 38-year-old woman with an episode of facial palsy 4 years ago, which responded to steroid treatment, presents again with left-sided facial nerve paralysis.

1. Which of the following disorders should be included in the differential diagnosis? (Choose all that apply.) A. Meningioma B. Schwannoma C. Hemangioma D. Lipoma E. Metastasis

2. Which of the following segments of the facial nerve is involved? A. Canalicular segment B. Labyrinthine segment C. Geniculate segment D. Tympanic segment E. Mastoid segment

3. All of the following statements about facial nerve hemangiomas are true EXCEPT: A. Honeycomb appearance is present in only half of

the cases. B. Honeycomb appearance is secondary to bone

spicules. C. The most common location is the mastoid segment

of the facial nerve. D. Hearing loss may accompany facial paresis. E. Histopathology shows a vascular malformation,

not hemangioma.

4. The facial nerve function significantly improves after surgical resection. True or false?

Page 100: Head and Neck Imaging - Case Review Series - Challenge

404

A N S W E R S

C A S E 1 9 9

Hemangioma of the Facial Nerve1. A, B, C, and E

2. C

3. C

4. False

ReferenceFriedman O, Neff BA, Willcox TO, et al. Temporal bone

hemangiomas involving the facial nerve. Otol Neu-rotol. 2002 Sep;23(5):760–766.

Cross-ReferenceNeuroradiology: THE REQUISITES, 3rd ed, p 403.

CommentHemangiomas in the temporal bone are rare lesions. They have a distinct tendency to occur in the region of the geniculate ganglion of the facial nerve, as seen in this case, presumably secondary to the abundance of the venous plexus around the facial nerve in this region. The origin of these lesions is not clear, but they probably reflect a form of venous malformation rather than true hemangioma; they do not arise from within the facial nerve but can infiltrate the nerve. They are called facial nerve hemangiomas because of their characteristic loca-tion and presentation with facial palsy. They typically are seen in young women. About 50% of facial hemangiomas show a pattern of trabecular bone on CT that has been likened to honeycomb (see the figures). With this char-acteristic appearance the diagnosis is straightforward. Schwannomas of the facial nerve can occur anywhere along the nerve, including the geniculate segment, and mimic hemangiomas. Meningiomas of the floor of the middle cranial fossa are also in the differential. Occasion-ally, facial nerve hemangiomas can attain large sizes and extend extradurally into the middle cranial fossa, mim-icking dural-based masses.

Hemangiomas can occasionally occur in the IAC; these do not demonstrate the honeycomb pattern and are almost invariably mistaken for schwannomas, which are abundant.

Notes

Page 101: Head and Neck Imaging - Case Review Series - Challenge

405

C A S E 2 0 0

History: A 6-year-old girl with a history of sickle cell anemia presents with sudden-onset bilateral orbital swelling, abdominal pain, and extremity pain.

1. In which compartment of the orbit are these lesions located? A. Intraconal B. Lacrimal gland C. Extraconal D. Subperiosteal E. Intraosseous

2. Which of the following defines orbital compression syndrome? A. A rare complication of sickle cell disease B. Optic nerve compression by enlarged extraocular

muscles C. Optic nerve compression by subperiosteal

abscesses D. Optic nerve compression by diffuse lymphoma

3. Bone infarcts in sickle cell anemia can occur in all of the following locations EXCEPT: A. Mandible B. Maxilla C. Orbit D. Vertebra E. Calvarium

4. Orbital compression syndrome is associated with epi-dural hematomas. True or false?

Page 102: Head and Neck Imaging - Case Review Series - Challenge

406

A N S W E R S

C A S E 2 0 0

Orbital Compression Syndrome1. D

2. A

3. B

4. True

ReferenceOzkavukcu E, Fitoz S, Yagmurlu B, et al. Orbital wall

infarction mimicking periorbital cellulitis in a patient with sickle cell disease. Pediatr Radiol. 2007 Apr;37(4):388–390. Epub 13 Feb 2007.

Cross-ReferenceNeuroradiology: THE REQUISITES, 3rd ed, p 335.

CommentVaso-occlusive syndrome of sickle cell anemia involving the orbit is a rare occurrence; the most commonly involved sites include the anterior chamber and fundus. Infarct of the bony orbit is even rarer, typically present-ing with marked diffuse swelling of the periorbital tissues mimicking periorbital cellulitis. Imaging studies may show a subperiosteal fluid collection or hematoma (see the figures) and compression of the globe, extraocular muscles, and optic nerves, hence the name orbital com-pression syndrome. Differentiation from periorbital cel-lulitis and subperiosteal abscess is required. Absence of paranasal sinus inflammatory changes is an important observation because most orbital inflammatory processes are secondary to sinusitis. Subperiosteal space in the orbit is a potential space that can be viewed as the orbital counterpart of the epidural space. Subperiosteal collec-tions in the orbit have convex margins similar to epidural hematomas because they are limited by the periosteum and cannot freely spread in the extraconal orbit. Subperi-osteal location of an orbital lesion places the process in the bony orbit or paranasal sinuses. Demonstration of bone infarcts by MRI or bone scan is diagnostic in this clinical setting. Treatment is conservative; surgical pro-cedures should be avoided unless there is optic nerve compromise.

Notes